Tải bản đầy đủ (.docx) (147 trang)

TAI LIEU BOI DUONG HOC SINH GIOI 9 PHAN DAI SO

Bạn đang xem bản rút gọn của tài liệu. Xem và tải ngay bản đầy đủ của tài liệu tại đây (3.33 MB, 147 trang )

<span class='text_page_counter'>(1)</span>PhÇn I : §¹i sè Các chuyên đề về biến đổi biểu thức Chuyên đề 1. Biến đổi biểu thức nguên. A. Một số hằng đẳng thức cơ bản.  (a + b)2 =a2 + 2ab + b2 .  (a + b + c )2 = a2 + b2 + c2 + 2ab + 2ac + 2bc  (a1 + a2 +...+ an )2 = a12 + a22 + ...+ an2 + + 2( a1a2 + ...+ a1an + a2a3 + ... + a2an + ... + an-1an ) n− 1. = a12 + a22 + ...+ an2 + 2 ∑. n. ∑. i=1 j=i+ 1. . ai a j. xn - yn = (x - y)(xn-1 + xn-2y + ... + xyn-2 + yn-1 ) ; n nguyªn d¬ng . x2k - y2k = (x + y)(x2k-1 - x2k-2y + x2k-3y2 - ... - y2k-1 ) ; k nguyªn d¬ng . x2k + 1 - y2k + 1 = (x + y)(x2k - x2k-1y + x2k-2y2 - ... + y2k ) ; k nguyªn d¬ng . ( x + y )3 = x3 + 3x2y + 3xy2 + y3 ( x - y )3 = x3 - 3x2y + 3xy2 - y3.    . B. B¶ng c¸c hÖ sè trong triÓn khai (x + y )n - Tam gi¸c Pascan §Ønh Dßng 1 ( n = 1 ) Dßng 2 ( n = 2 ) Dßng 3 ( n = 3 ) Dßng 4 ( n = 4 ) Dßng 5 ( n =5 ). I.. VÝ dô. 1. 1. 1 5. 1 4. 1 3 10. 1 2 6. 1 3 10. 1 4. 1 5. 1. 1.  VÝ dô 1 Chøng minh r»ng : 3 a ¿ ( a+ b+c ) =a3 +b3 +c 3 +3 [ a 2 ( b+ c )+ b2 ( a+c ) +c 2 ( a+b ) ]+6 abc. ¿ +3 [ a2 ( b+c +d ) +b 2 ( a+c +d ) +c 2 ( a+b+ d )+ d 2 ( a. b ( a+ b+c +d ) ¿3=a3 +b3 +c 3 +d 3 ¿ ¿ a=b=c ¿ a +b + c = 0 ¿ ¿ ¿ 3 3 c a ¿ +b + c3 =3 abc ⇔¿ ¿. Gi¶i a). Biến đổi vế trái ta có :.

<span class='text_page_counter'>(2)</span> 3. ( a+b+ c )3=[ ( a+ b ) +c ] =( a+ b )3 +3 ( a+ b )2 c+3 ( a+ b ) c 2+ c 3 = a3 +b 3+3 a2 b +3 ab2 +3 a2 c+ 3 b2 c+3 ( a+b ) c 2 +c 3+ 6 abc 3 3 3 2 2 2 =a +b + c +3 [ a ( b+ c )+ b ( a+c ) +c ( a+b ) ]+ 6 abc. VËy : ( a+b +c )3=a 3+ b3 +c 3 +3 [ a2 ( b+c ) +b 2 ( a+ c )+ c 2 ( a+ b ) ] +6 abc. (®pcm) b) Chøng minh t¬ng tù c©u a c) Ta cã : 3. 3. 3. 3. 3. a + b +c − 3 abc=( a+b ) +c − 3 ab(a+ b)− 3 abc 2 = ( a+b+ c ) [ ( a+ b ) − ( a+ b ) c+ c 2 ] − 3 ab ( a+b+ c ) = ( a+b+ c ) (a2 +b2 + c2 −ab − ac − bc) 1 = ( a+b+ c ) [ ( a+ b )2 + ( b+ c )2 + ( a+ c )2 ] 2 Vậy điều kiện cần và đủ để : a3 +b 3+ c 3=3 abc là. - HoÆc a + b + c = 0 2. 2. 2. - HoÆc (a + b) + (b + c) + (a + c) = 0 Û a = b = c  VÝ dô 2 Ph©n tÝch ®a thøc thµnh nh©n tö a) a2(b - c) + b2(c - a) + c2(a - b) . b) a3(b - c) + b3(c - a) + c3(a - b) . c) x3 - 3(a2 + b2)x + 2(a3 + b3) : Gi¶i a a ¿2 ( b − c ) +b 2 ( c − a ) +c 2 ( a− b ) =a2 ( b −c ) −b 2 [ ( a − b ) + ( b − c ) ] + c2 ( a −b ) ¿ b a ¿3 ( b − c )+ b3 ( c − a ) +c 3 ( a −b )=a3 ( b − c ) −b 3 [ ( a− b ) + ( b − c ) ] +c 3 ( a− b ) ¿. c) §Æt S = a + b vµ P = ab Ta cã : a2 + b2 = (a + b)2 - 2ab = S2 - 2P ; a3 + b3 =(a + b)3 -3ab(a + b) = S3 - 3SP . V× vËy x3 - 3(a2 + b2)x + 2(a3 + b3) = x3 - 3(S2 - 2P)x + 2(S3 -3SP) = (x3 -3S2x + 2S3) + 6P(x -S) =(x - S)(x2 + Sx - 2S2) + 6P(x -S) =(x - S)( x2 + Sx - 2S2 +6P) =(x - a - b)[x2 + (a + b)x - 2(a2 + b2 - ab)]. II. Bµi tËp 1.. a. b. c. d. e. f. g. h.. Ph©n tÝch ®a thøc thµnh nh©n tö : x3 + 4x2 - 29x + 24 . x4 + 6x3 + 7x2 - 6x + 1 . 6x5 + 15x4 + 20x3 + 15x2 + 6x +1 . x8 + x4 +1 . x10 + x5 + 1 . x12 + 1 . x6 + 3x5 + 4x4 + 4x3 + 4x2 + 3x + 1 . (a + b + c)3 - a3 - b3 - c3 ..

<span class='text_page_counter'>(3)</span> 2.. 3.. i. j. k. a. b.. (a - b)3 + (b - c)3 + (c - a)3 . (x2 - x + 2)2 + (x - 2)2 . (x + y + z )5 - x5 - y5 - z5 . §¬n gi¶n biÓu thøc (x + y + z)3 - (x + y - z)3 - (y + z - x)3 - (z + x - y)3 . ( 2 + 1)(22 + 1)(24 + 1)(28 + 1)(216 + 1)(232 + 1) . Ba sè a , b , c tho¶ m·n ®iÒu kiÖn. ¿ a+ b+c=0 a2 +b 2+ c 2=14 ¿{ ¿. TÝnh A = a4 + b4 + c4 . 4. Hai sè a , b lÇn lît tho¶ m·n c¸c hÖ thøc sau a3 -3a2 + 5a -17 = 0 vµ b3 - 3b2 + 5b +11 = 0 . H·y tÝnh a + b . 5. Cho a3 - 3ab2 = 19 ; b3 -3a2b = 98 . TÝnh P = a2 + b2 . 6. Cho a2 + b2 + c2 = a3 + b3 + c3 = 1 . TÝnh a2 + b9 + c1945 . 7. Cho x + y + z = 0 . Chøng minh r»ng : a. 2(x5 + y5 + z5 ) = 5xyz(x2 + y2 + z5) . b. x7 + y7 + z7 = 7xyz(x2y2 + y2z2 + z2y2 ) . c. 10(x7 + y7 + z7) = 7(x2 + y2 + z5) (x5 + y5 + 5 z ). 8. Cho c¸c sè a , b, c , d tho¶ m·n a2 + b2 + (a + b)2 = c2 + d2 + (c + d)2 Chøng minh r»ng a4 + b4 + (a + b)4 = c4 + d4 + (c + d)4 . 9. Chøng minh r»ng nÕu c¸c sè a , b , c , d tho¶ m·n a2 + b2 + (a - b)2 = c2 + d2 + (c - d)2 . Th× a4 + b4 + (a - b)4 = c4 + d4 + (c - d)4 .. Chuyên đề 2. Biến đổi phân thức hữu tỷ. I.VÝ dô :  VÝ dô 1 : ba sè thùc kh¸c kh«ng a , b , c tho¶ m·n ®iÒu kiÖn a + b + c ¹ 0 vµ 1 1 1 1 + + = a b c a+b+ c. Chứng minh rằng trong ba số a , b , c có hai số đối nhau . Từ đó suy ra mọi số nguyên lẻ ,thì 1 1 1 1 + n+ n= n n n n a b c a +b + c. Gi¶i. Tacã: 1 1 1 1 1 1 1 1 a b a b    Û    Û  a b c a b c a b a b c c ab c (a  b  c ) Û ( a  b)(ac  bc  c 2 )  (a  b)ab Û ( a  b)( ac  bc  c 2  ab) 0.

<span class='text_page_counter'>(4)</span>  a -b Û (a  b)(a  c)(b  c) 0 Û  b -c  c -a n n a =−b ¿ b n=− cn ¿ 1 1 1 1 + n+ n= n n n VËy nÕu n lÎ th× c n=− an n a b c a +b + c ¿ ⇒ ¿ ¿.  VÝ dô 2 : Rót gän biÓu thøc : a+b ¿3 ¿ a+b ¿ 4 ¿ a+b ¿5 ¿ ¿ ¿ ¿ 1 A= ¿. Gi¶i : §Æt S = a + b vµ P = ab . Þ a2 + b2 = (a + b)2 - 2ab = S2 - 2P a3 + b3 = (a + b)3 - 3ab(a + b) = S3 - 3SP . VËy : 1 1 a b S    a b ab P 2 1 1 a  b2 S 2  2P   2 2  a 2 b2 ab P2 2 1 1 a3 + b3 S (S − 3 P) + = = a3 b3 a3 b3 P3 2 2 1 S(S −3 P) 3 S −2 P 6 S ⇒ A= 3 ⋅ + ⋅ + 5⋅ S P3 S4 P2 S P 5 1 S 1 1 3 2 2 2 = 5 3 [ S ( S −3 P)+ 3SP(S −2 P)+ 6 SP ]= 5 3 = 3 = 3 3 S P S P P a b.  VÝ dô 3 Cho ba sè a , b , c ph©n biÖt . Chøng minh r»ng gi¸ trÞ cña biÓu thøc sau kh«ng phô thuéc vµo gi¸ trÞ cña biÕn x : S ( x)=. ( x − a)(x − b) (x −b)( x −c ) (x −a)(x −c ) + + (c − a)(c −b) (a −b)( a− c) (b −a)(b − c). Gi¶i : §Æt P(x) = S(x) - 1 th× ®a thøc cã bËc kh«ng vît qu¸ 2 . mÆt kh¸c ta thÊy : P(a) = P(b) = P(c) = 0.

<span class='text_page_counter'>(5)</span> Tøc lµ a , b , c lµ ba nghiÖm ph©n biÖt cña P(x) ®iÒu nµy chØ x¶y ra khi khi ®a thøc P(x) lµ ®a thøc kh«ng , tøc lµ P(x) = 0 víi mäi x suy ra S(x) = 1 . VËy gi¸ trÞ cña biÓu thøc S(x) kh«ng phô thuéc vµo gi¸ trÞ cña x .. II Bµi tËp: 1. Rót gän biÓu thøc. 1999 1999 1999 1999 1+ 1+ 1+ ⋅⋅ (1+ ( )( )( ) 1 2 3 1000 ) A= 1000 1000 1000 1+ 1+ ⋅⋅ (1+ ( 1+ 1000 )( )( ) 1 2 3 1999 ) 2 n− 1 ¿2 (¿¿) 4 , víi n ³ 1 1− ¿ 4 4 4 B= 1− 1− 1− ⋅⋅¿ 1 9 25 y − x y+ x 2 y +x 2 C=2 + − + + xy xy z xy z 2 x − 25 x 2 −25 y= ; z= Trong đó x > 5 và 10 x +25 15 x+ 25 x+ x+ x x −5 k k k a b c D= + + ( a −b)(a − c)(x −a) (b − a)(b −c )(x − b) (c − a)( c − b)(x − c). ( )( )(. | ||. ). |. ứng với k = 0 , 1 , 2 , 3 và a , b , c đôi một khác nhau . k. E=. k. k. ứng với k = 0 , 1 , 2 , 3 và a , b , c , d đôi một khác nhau .. (. k. a b c d + + + (a −b)( a− c)(a −d ) ( b −a)(b − c)(b − d ) (c −a)(c − b)(c − d) (d −a)(d − b)(d − c). a+b ¿3 ¿ a+ b ¿4 ¿ a+b ¿5 ¿ 1 1 − 2 . 2 a b ¿ ¿ ¿ ¿ 1 F= ¿. ). 3 2 2. Cho ph©n thøc P= 3n +22n − 1 .. n +2 n +2 n+1. a. H·y rót gän ph©n thøc trªn . b. Chøng minh r»ng nÕu n lµ mét sè nguyªn th× gi¸ trÞ cña ph©n thức tìm đợc trong câu a. tại n luôn là một phân số tối giản ..

<span class='text_page_counter'>(6)</span> 2. Cho c¸c sè kh¸c kh«ng a , b , c tho¶ m·n ®iÒu kiÖn : a + b + c = 0 . 2. 1 1 1 1 1 1 + 2 + 2= + + 2 a b c a b c ¿ a+ b+c=2001 3. Cho ba sè thùc a , b , c tho¶ m·n 1 + 1 + 1 = 1 a b c 2001 ¿{ ¿. (. Chøng minh r»ng :. ). Chøng minh r»ng Ýt nhÊt mét trong ba sè a , b , c b»ng 2001 . 4. Cho a , b , c Î R chøng minh r»ng ( a −b )5 + ( b − c )5+ ( c − a )5 ( a −b )3+ ( b −c )3 + ( c −a )3 ( a− b )2 + ( b − c )2+ ( c − a )2 = ⋅ . 5 3 2. 5. Cho c¸c sè nguyªn kh«ng ©m k1 , k2 , ...,kn (n lµ sè nguyªn d¬ng ) tho¶ m·n ®iÒu kiÖn : k1 + k2 + ... + kn lµ mét sè lÎ . Chøng minh r»ng nÕu c¸c sè a1 , a2 ,...,an tho¶ m½n :. |a1 − a2| |a2 −a3| =. k1. k2. =⋅=. |an − a1| kn. th× a1 = a2 = ...= an .. 6. Cho ba sè kh¸c nhau a , b , c . a. Chứng minh rằng khi k = 0 , 1 , 2 thì ta có hằng đẳng thức a. k. (x − b)( x − c) k (x −a)( x −c ) k ( x −b)(x −a) k +a +a =x . (a− b)(a − c) (b −a)( a− c) (c −b)(c − b). b. Hằng đẳng thức trên còn đúng không nếu thay k = 3 ? . a b c + + =0 . 7. Cho ba sè a , b , c lµ ba sè kh¸c nhau vµ b− c c −a a −b Chøng minh r»ng a b c + + =0 2 2 ( b− c ) ( c − a ) ( a −b )2. .. 8. Ba sè a , b , c kh¸c nhau vµ kh¸c 0 tho¶ m·n ®iÒu kiÖn a + b + c = 0 Chøng minh r»ng b −c c − a c a b + + + + =9 ( a −b )( c a b a−b b−c c−a). .. 1 9. Cho a ¹ 0 vµ a+ a lµ mét sè nguyªn . Chøng minh r»ng víi mäi sè 1 nguyªn n th× an + n lµ mét sè nguyªn . a. 10.a. Cho a > b > 0 , n Î N * . So s¸nh hai sè A vµ B : 2. A=. n −1. 1+a+ a +⋅+ a 2 n 1+ a+a +⋅+a. 2. ; B=. n −1. 1+b+ b +⋅+b 2 n 1+ b+b +⋅+b. b. So s¸nh hai sè C vµ D ( cã 10 ch÷ sè 0 sau mçi dÊu phÈy ) :.

<span class='text_page_counter'>(7)</span> 2. 1 ,00000000004 ¿ + 2, 00000000002 ¿ 1 , 00000000002¿2 +2 , 00000000002 ¿ ¿ 2 , 00000000004 C= ¿. 11. a. Cho các số a , b , c đôi một phân biệt đặt : ¿ a bk ck S k= + + , k∈ ( a −b)( a− c) (b − a)(b −c ) (c − a)(c − b) ¿ k. N.. TÝnh S0 , S1 , S2 ,S3 . b. Cho ba số a , b , c đôi một khác nhau đặt : T k =a k. ( a+b)(a+ c) k (b+ a)(b+c ) k (c +a)(c+b) +b +c ( a− b)(a − c) (b − a)(b −c ) (c −a)(c −b). .. TÝnh T0 , T1 , T2 . 12.Cho c¸c sè kh¸c kh«ng a , b , c . TÝnh gi¸ trÞ biÓu thøc 2003 2003 2003 . T =x + y + z BiÕt x , y , z tho¶ m·n c¸c ®iÒu kiÖn 2. 2. 2. 2. 2. 2. x + y +z x y z = 2+ 2 + 2 2 2 2 a + b +c a b c. 13.Cho c¸c sè a , b , c , x , y , z tho¶ m·n :. .. ¿ x=by+ cz y=cz+ax z=ax + by ¿{{ ¿. BiÕt r»ng a , b , c kh¸c -1 . TÝnh gi¸ trÞ cña biÓu thøc sau : 1 1 1 M= + + .. 1+ a 1+b 1+c 1 14.Cho x > 0 tho¶ m·n ®iÒu kiÖn x 2+ 2 =7 x 1 5 N=x + 5 . x. Chuyên đề 3. . TÝnh gi¸ trÞ cña biÓu thøc :.

<span class='text_page_counter'>(8)</span> I.. Biến đổi biểu thức có chứa căn thức. Mét sè kiÕn thøc c¬ b¶n 1. C¨n bËc hai . Mỗi số dơng a > 0 có hai căn bậc hai là hai số đối nhau : √ a > 0 gäi lµ c¨n bËc hai sè häc hay c¨n bËc hai d¬ng cña a vµ - √ a < 0 lµ c¨n bËc hai ©m cña a .  Sè 0 cã c¨n bËc hai duy nhÊt lµ 0 .  Sè ©m kh«ng cã c¨n bËc hai .  Quy íc : sau nµy , nÕu kh«ng nãi g× thªm th× ta hiÓu r»ng “ c¨n bËc hai cña sè a > 0 “ lµ c¨n bËc hai d¬ng cña a .. 2. C¨n bËc n ( n Î N , n³ 2 ) a. §Þnh nghÜa : C¨n bËc n ( n Î N , n³ 2 ) cña mét sè a lµ mét sè thùc b (nÕu cã) sao cho bn = a . b. Chó ý :  Đối với căn bậc lẻ (n = 2k + 1): mọi số đều có căn bậc hai lẻ vµ chØ cã mét c¨n bËc hai lÎ . C¨n bËc hai lÎ cña sè d¬ng lµ sè d¬ng , cña sè 0 lµ sè 0 , cña sè ©m lµ sè ©m . Ký hiÖu 2 k+1 √a  §èi víi c¨n bËc hai ch½n (n = 2k) : sè ©m kh«ng cã c¨n bËc hai ch½n . sè 0 cã c¨n bËc hai ch½n lµ 0 . Sè d¬ng cã hai c¨n bậc hai chẵn là hai số đối nhau ký hiệu là 2√k a và - 2√k a (trong đó 2√k a ³ 0). 3. Một số phép biến đổi căn thức cơ bản a. Biến đổi căn thức bậc lẻ 2 k 1. A2 k 1  A. 2 k 1. AB 2 k 1 A 2 k 1 B. 2 k 1. A 2 k 1 A  ,B¹ 0 B 2 k 1 B. 2 k 1. A2 k 1 B  A2 k 1 B. b. Biến đổi căn thức bậc chẵn 2k 2k. √ A2 k =| A|. 2k. 2k. √ A ⋅ B= √| A| √|B| , AB ≥ 0. √. 2k. 2k. 2k A √| A| = 2k , AB ≥ 0,B≠ 0 B √|B|. √ A 2 k ⋅B=| A|2√k B , B≥ 0. Đẳng thức sau thờng đợc sử dụng trong các phép biến đổi căn thức m n √ √ A=mn√ A , A ≥ 0.

<span class='text_page_counter'>(9)</span> II.. c. Chú ý : Trong các biến đổi vừa nêu k , m , n là những số nguyên d¬ng. Mét sè vÝ dô.  VÝ dô 1 Chøng minh r»ng 3 3. √ √2 −1=. 1 32 34 − + 9 9 9. √ √ √ 3. Gi¶i : Đặt √3 2=a thì a3 = 2 đẳng thức cần chứng minh là 1 −a+a2. 3. √ a −1= 3 √9 Ta cã 3 = 2 + 1 = a3 + 1 = (a + 1)(a2 - a + 1) . 1 = 2 - 1 = a2 - 1 = (a - 1)(a2 + a + 1) . Biến đổi vế trái ta có : 1+a ¿3 ¿ 3 ¿= 3 3 2 a +3 a +3 a+1 ¿ VËy : 3 ¿ ¿ 3 2 1 − a+a 3 3 3 √ =3 = =√ ¿ 3 √9 √ 9(1+a) 1+a. √. 3. √ a −1=. 1 −a+ a2 √3 9. tøc lµ. 3 3. √ √2 −1=. 1 32 34 − + 9 9 9. √ √ √ 3. (®pcm) ..  VÝ dô 2 Cho hai sè d¬ng a vµ b . Chøng minh r»ng. Gi¶i. 2. Ta cã : 2( ab]. 2. 2. 2. 2. √ a2 +b2 -a)( √ a2 +b2 -b) = 2[a + b - (a + b) √ a2 +b2 + =(a2 + 2ab + b2 ) - 2(a + b). b2). 2. b − √ a +b √ 2(√ a +b −a)( √a + b2− b)=a+ 2. √ a2 +b2 + (a2 +. =(a + b)2 -2(a + b) √ a2 +b2 + (a2 + b2) =(a + b - √ a2 +b2 )2 Vì a , b đều dơng nên (a + b)2 = a2 + 2ab + b2 > a2 + b2 Þ a + b > √ a2 +b2 VËy: √ 2( √ a2 +b2 −a)( √a 2+ b2 − b)=a+ b − √ a2 +b 2  VÝ dô 3 Chøng minh nÕu |x|≥| y| th× |x + y|+|x − y|=|x+ √ x 2 − y 2|+|x − √ x 2 − y 2| Gi¶i : §Æt A=| x+ y|+|x − y| th× A ³ 0 vµ.

<span class='text_page_counter'>(10)</span> x − y ¿2 +2|x 2 − y 2| ¿ Tõ gi¶ thiÕt ta x − y ¿2 +¿ ¿ 2 2 2 A =|x + y| +|x − y| +2|x + y|⋅|x − y|=¿ cã x2 ³ y2 nªn |x 2 − y 2|=x 2 − y 2 . VËy. A2 = 2(x2 + y2) + 2(x2 - y2) = 4x2 Þ A = 2| x|. (1). (2). Nh vËy víi mäi sè y mµ x2 ³ y2 th× sè A kh«ng phô thuéc vµo y vµ A = 2| x| . §Æt z=√ x 2 − y 2 Þ x2 ³ z2 vËy : |x + √ x 2 − y 2|+|x − √ x 2 − y 2| = 2|x| Từ (1) , (2) và cách đặt A suy ra : |x + y|+|x − y|=|x+ √ x 2 − y 2|+|x − √ x 2 − y 2|  Ví dụ 4 Với mỗi k nguyên dơng đặt : k k S k =( √ 2+1 ) + ( √ 2 −1 ) . Chøng minh r»ng víi mäi sè nguyªn d¬ng m , n ( m > n) th× Sm + n + Sm - n =SmSn .. Gi¶i. ¿. §Æt. III. Bµi. tËp. x 1= √ 2+1 x 2=√ 2− 1 th× x1x2 = 1 ¿{ ¿. VËy víi mäi sè nguyªn d¬ng m , n ( m > n ) th× Sm + n + Sm - n = x1m + n + x2m + n + x1m - n + x2m - n = x1m + n + x2m + n + x1nx2n(x1m - n + x2m - n ) = x1m + n + x2m + n + x1mx2n + x1nx2m = (x1m + x2m)(x1n + x2n) = Sm Sn VËy : Sm + n + Sm - n =SmSn (®pcm) .. 1.Rót gän biÓu thøc : A= √ 13 − √160 − √ 53+ 4 √ 90 B=√ 13+30 √2+ √ 9+ 4 √ 2 C=√ 8+2 √10+2 √ 5+ √ 8 −2 √10+ 2 √ 5.

<span class='text_page_counter'>(11)</span> 1 + √175 −2 √ 2 √ 8+ √7 E= √ 0 , 25 √ 961+2 √ 10+ √15+ √ 6 a 3 − 3 a+(a2 − 1) √ a2 −4 −2 F= 3 a −3 a+(a2 −1) √a 2 − 4+ 2 a+b ¿2 ¿ 2 a +b2 ¿2 ¿ ¿ ¿ ¿ 1 1 1 + + a4 b4 ¿ ¿ 1 1 + 2 2 ¿ a +b M =√ ¿ D=. 2. TÝnh gi¸ trÞ cña c¸c biÓu thøc sau: A= √ a+6+6 √ a −3+ √ a+6 −6 √ a −3 víi a ³ 3 B= 1+99 . .. 92 +0, 99 . .. 92 ⏟ ⏟ C=. √. √. n. n. 1 1 1 1 1 1 1 1 1 + 2 + 2 + 2 + 2 + 2 + ⋅+ 2 + 2 + 2 1 2 3 1 3 4 1 99 1002. D=2 x 3+ 2 x 2 +1. √. Víi. x=. 1 3. (√ 3. √. 23+ √ 513 3 23 − √ 513 + −1 4 4. √. 3. a. Cho ( x + √ x2 +3 )( y+ √ y 2 +3 ) =3 . TÝnh E = x + y. 3 1 b. Cho x=√ √ 2−1 − 3 . TÝnh F = x3 + 3x + 2 . √ √ 2−1 c. TÝnh tæng N = a1 + a2 + ...+ a99 . Víi :. 1 , n = 1,⋅⋅ ,99 . (n+1) √n+ n √ n+1 x −1 −2 √ x − 2 A= √ √ x − 2− 1 an =. 4. a. Cho. a.1) Tìm điều kiện của x để A có nghĩa . a.2) Rót gän A . 1 √ x +1 b. Cho B= 2 : . x √ x + x +√ x x −√ x b.1) Tìm điều kiện của x để B có nghĩa . b.2) Rót gän B . c. Cho c¸c biÓu thøc 3 2 x − 3 √ x −2 vµ D= √ x − √ x +2 x −2 . C= √ x −2 √ x +2 c.1) Rót gän C vµ D .. ).

<span class='text_page_counter'>(12)</span> c.2) Tìm các giá trị của x để C = D . d. Cho E=3a + √ 9a − 3 − √ a −2 + 1 −1 . a+ √ a −2 √ a −1 √ a+2 5.Chứng minh đẳng thức : 3 3 1+ √ 1− √ 2. a.. 2. +. √3 1 − 1 − √3 1+ 1+. √. √. Tìm a để |E|=1. .. =1. 2 2 2 2 2 a − √a a + √a b. 2 − 2 +a+ 1=( √ a −1 ) víi a ³ 0 . a + √ a+1 a − √a+ 1 c. √ a+b+ c+ 2 √ ac+ bc + √ a+b+ c − 2 √ ac+ bc=¿ 2 √ a+b nÕu a+b ≥ c ¿ 2 √ c nÕu a+ b < c ¿ ¿ ¿ ¿. Víi a ,b , c lµ ba sè d¬ng .. 2 nÕu 1≤ a ≤ 2 ¿ 2 √ a-1 nÕu a >2 ¿ ¿ ¿ √ a+2 √ a −1+ √ a −2 √ a −1=¿. d.. 847 3 847 + 6− =3 27 27. √√ √ √ 3. e.. 6+. .. 6. Chøng minh r»ng 2 − √ 2+ √ 2+ √ 2+ √2+⋅+ √ 2 > 1 . ( ë vÕ tr¸i tö cã 4 2− √2+ √ 2+ √ 2+⋅ √ 2 n dÊu c¨n , mÉu cã n -1 dÊu c¨n . 7. a. Cho a , b , c , d , A , B , C , D lµ nh÷ng sè d¬ng vµ a b c d = = = .Chøng minh r»ng A. B. C. D √ aA+ √ bB+ √ cC+√ dD=√( a+b+c +d )( A+ B+C + D). 3 3 3 3 y 2+ y 2 + √ x 2 y 4 =a , Chøng minh : √ x 2+ √ y 2 =√ a2 . ¿ 3 3 3 ax =by =cz 1 1 1 + + =1 c. Cho . x y z 3 3 3 2 2 2 3 . Chøng minh r»ng : √ ax + by +cz =√ a+ √ b+ √c ¿{ ¿ d. Chøng minh r»ng nÕu √3 a+ √3 b+ √3 c= √3 a+b+c th× víi mäi sè nguyên dơng lẻ n , ta đều có : √n a+√n b+ √n c= √n a+b+c .. b. Cho. 2. 3. √ x +√ x. 4. √.

<span class='text_page_counter'>(13)</span> 8. a. Chøng minh r»ng víi a> 1 th× sè sau ®©y lµ mét sè 8 nguyªn : a+1 8 a −1 3 a+1 8 a −1 + a− 3 3 3 3. √ √ √ √ √√ √ √ 3. x= a+. .. b. Chøng minh r»ng x= 3 3+ 9+ 125 + 3 −3+ 9+125 , lµ mét sè 27 27 nguyªn . c. Cho A lµ mét tËp con cña tËp c¸c sè thùc R tho¶ m·n : A É Z, √ 2+ √ 3 ∈ A , nÕu x , y ∈ A th× x + y vµ xy Î A . Chøng minh r»ng : √ 2− √ 3 ∈ A . 9. a. Chøng minh r»ng ph¬ng tr×nh x5 + x +1 = 0 cã nghiÖm duy nhÊt lµ : 1 3 25+ √ 621 3 25 − √ 621 . x= 1− − 3. ( √. 2. √. 2. ). b. Chøng minh r»ng x0 = √ 2+ √2+ √ 3 − √ 6 −3 √ 2+ √ 3 lµ mét nghiÖm cña ph¬ng tr×nh : x4 - 16x2 + 32 = 0 . c. Chứng minh rằng x=√3 9+ 4 √5+ √3 9− 4 √ 5 là nghiệm của phơng trình x3 - 3x -18 = 0 . Từ đó hãy tìm x . 10.a. Chøng minh mçi sè h¹ng cña d·y sè a1 , a2 , ... , ak , ...víi n n ( 2+ √ 3 ) − ( 2− √ 3 ) lµ mét sè nguyªn .T×m tÊt c¶ c¸c an = 2 √3 giá trị của n để an chia hết cho 3 . b. Chøng minh r»ng mçi sè h¹ng cña d·y sè d1 , d2 , ... , dk , ... n n 3+ √ 5 3 − √5 víi d = + −2 lµ mét sè tù nhiªn .T×m tÊt c¶ c¸c n. (. 2. )(. 2. ). giá trị của n để dn là một số chính phơng .. Các chuyên đề về phơng trình Chuyên đề 1 Ph¬ng tr×nh ®a thøc mét Èn-§Þnh lý viÐt A. Ph¬ng tr×nh ®a thøc mét Èn I. KiÕn thøc c¬ b¶n 1.Ph¬ng tr×nh bËc nhÊt mét Èn : §Þnh nghÜa. a. Ph¬ng tr×nh Ax + B = 0 . Trong đó x là ẩn , A và B là những số thực hoặc biểu thức có chứa tham số , đợc gọi là phơng trình bậc nhất một ẩn x ..

<span class='text_page_counter'>(14)</span> b.. C¸ch gi¶i :.  NÕu A ¹ 0 th× ph¬ng tr×nh cã nghiÖm duy nhÊt lµ x=− B . A  NÕu A = 0 vµ B = 0 th× tËp nghiÖm cña ph¬ng tr×nh lµ R .  NÕu a = 0 vµ B ¹ 0 th× ph¬ng tr×nh v« nghiÖm . Chó ý : NÕu A hoÆc B lµ nh÷ng biÓu thøc kh¸ cång kÒnh hoÆc chøa nhiều tham số thì phải tinh ý xem phơng trình đã cho có phải là dạng bậc nhất đối với ẩn hay không . c. : . VÝ dô VÝ dô 1 Gi¶i ph¬ng tr×nh. x +1 x +2 x+3 x+ 4 + = + (1) . 58 57 56 55  x 1   x  2   x  3   x  4  Gi¶i : (1) Û  1    1    1   1  58   57   56   55  x  59 x  59 x  59 x  59 Û    58 57 56 55 1 1 1 1 ⇔ ( x +59 ) + − − =0 58 57 56 55 ⇔ x +59=0 ⇔ x=-59. (. ). VËy ph¬ng tr×nh cã nghiÖm duy nhÊt lµ x = - 59 .  Gi¶i ph¬ng tr×nh. VÝ dô 2. ¿ a+b − x b +c − x c +a − x 4x + + + =1 (2) . c a b a+ b+c 1 1 1 4 Víi : a , b , c ≠ o ; a +b+ c ≠ 0 vµ + + ≠ ⋅ a b c a+ b+c ¿ a+ b − x b+ c − x c+ a − x 4x Gi¶i :(2) ⇔ +1 + +1 + +1 =4 − c a b a+ b+c a+b +c − x a+b+ c − x a+b+ c − x 4 (a+ b+c − x) ⇔ + + = c a b a+ b+c 1 1 1 4 ⇔ (a+b +c − x) + + − =0 a b c a+b+ c ⇔ a+b+ c − x=0 ⇔ x=a+ b+c VËy ph¬ng tr×nh (2) cã nghiÖm duy nhÊt lµ x = a+b +c. (. )(. (. . )(. ). ). VÝ dô 3 Gi¶i ph¬ng tr×nh :. x −a x − b x − c 1 1 1 1 1 1 + + =2 + + víi a , b , c ≠ 0 vµ + + ≠ 0 bc ac ab a b c ab ac bc. (. ). (3).

<span class='text_page_counter'>(15)</span> 1 1  b c   1  1 1 1  a Gi¶i : (3) Û     x 2           bc ac ab   a b c   bc ac ab  1 1   1 1 1 a  1 1 b  1 1 c  Û     x                bc ac ab   b c bc   c a ca   a b ab  1 1  a b c a b c a b c  1 Û    x   bc ac ab  bc ac ab  1 1  1 1   1  1 Û     x (a  b  c)      bc ac ab   ab ac bc  1 1   1 Û     [ x  ( a  b  c) ] 0  ab ac bc  Û x a  b  c. VÝ dô 4 Gi¶i ph¬ng tr×nh :. . x −ab x − bc x − ca 1 1 1 + + =a+b+ c ví i + + ≠ 0 (4) a+b b+c c +a a+b b+c c+ a 1 1 1 ab bc ca Gi¶i : (4) ⇔ + + x=a+b+c + + + a+ b b+ c c +a a+b b+ c c+ a 1 1 1 bc ca ab ⇔ + + x= a+ + b+ + c+ a+b b+c c+ a b+c c +a a+b 1 1 1 ab+ bc+ca ab+ bc+ca ab+ bc +ca ⇔ + + x= + + a+b b+c c+ a a+b b+c c+a 1 1 1 1 1 1 ⇔ + + x=( ab+ bc+ ca ) + + a+b b+c c+ a a+ b b +c c +a 1 1 1 ⇔ + + [ x − ( ab + bc+ca ) ]=0 a+b b+c c+ a ⇔ x=ab+ bc+ca. ( ( ( ( (. ) ) ( ) ) ). )(. )(. (. ). ). . Ph¬ng tr×nh bËc hai. 2. .§Þnh nghÜa : Ph¬ng tr×nh bËc hai lµ ph¬ng tr×nh cã d¹ng. a. ax2 + bx + c = 0 ( a ¹ 0) (1) . Trong đó a , b , c là những số thực đã cho , x là ẩn số . b. (1) 2 Biểu thức D = b - 4ac đợc gọi là biệt thức của phơng trình (1) . Ta xÐt c¸c trêng hîp : D < 0 th× ph¬ng tr×nh (1) v« nghiÖm . D = 0 th× ph¬ng tr×nh (1) cã mét nghiÖm kÐp : b x1 = x2 = − .. C«ng thøc nghiÖm ph¬ng tr×nh. 2a. D > 0 th× ph¬ng tr×nh cã hai nghiÖm ph©n biÖt : − b+ √ Δ − b −√ Δ . x 1= ; x 2= 2a. 3.. Mét sè ph¬ng tr×nh quy vÒ ph¬ng tr×nh bËc hai. 2a.

<span class='text_page_counter'>(16)</span> .Ph¬ng tr×nh d¹ng :. a. af2 (x) + bf(x) + c = 0 , trong đó f(x) là. hµm sè víi biÕn sè x . Đặc biệt khi f(x) = x2 phơng trình ax4 + bx2 + c = 0 đợc gọi là ph¬ng tr×nh trïng ph¬ng . a. §Æt t = f(x) chuyÓn ph¬ng tr×nh vÒ d¹ng at2 + bt + c = 0 Sau khi tìm đợc t , tìm x theo t . a.2. 1C¸ch gi¶i :. VÝ dô :  VÝ dô 5 : Gi¶i ph¬ng tr×nh :. x4 + 24x - 112 = 0 . (5). Gi¶i : §Æt t = x2 ³ 0 , ta cã (5) Û t2 + 24t -112 = 0 Û t = -28 (lo¹i) hoÆc t = 4 . Với t = 4 thì x = ± 2 . Vậy phơng trình đã cho có hai nghiÖm lµ : x = -2 vµ x = 2 . (x + a)4 +(x + b)4 = c .. Ph¬ng tr×nh d¹ng: b.1 C¸ch gi¶i : §Æt t =. b.. x+. a+ b 2. , ®a vÒ d¹ng ph¬ng tr×nh trïng. ph¬ng Èn t . b.2. VÝ dô :  VÝ dô 6 : Gi¶i ph¬ng tr×nh :. (x + 1)4 +(x + 5)4 = 40 . (6). Gi¶i : §Æt t = x + 3 , ta cã : (6) Û (t - 2)4 + (t + 2)4 = 40 Û t4 + 24t2 -4 = 0 Û t = ± √√ 148 −12 VËy x = ± √ √ 148 −12 - 3 :(x + a)(x + b)(x + c)(x + d) = m (*), víi a + b =c + d 2 Đặt t = x + (a + b)x + e , trong đó e là một số thực đợc chọn thích hợp. c.Ph¬ng tr×nh cã d¹ng c.1 c.2. C¸ch gi¶i :. VÝ dô :  VÝ dô 7 : Gi¶i ph¬ng tr×nh :. Gi¶i:. (x - 1)(x - 2)(x + 4)(x + 5) = 112 .(7). (7)⇔ [ ( x −1 ) ( x + 4 ) ][ ( x − 2 )( x +5 ) ]=112 ⇔ ( x 2 +3 x − 4 )( x 2 +3 x − 10 )=112 (7) 2 §Æt t= x +3 x − 7 ta cã : 2 (7)⇔( t - 3)(t + 3)= 112 ⇔t =121 ⇔ t=±11. +)NÕu t = -11 th× x2 +3x -7 = -11Û x2 +3x + 4 = 0 ,v« nghiÖm +) NÕu t = 11 th× x2 +3x -7 = 11Û x2 +3x -18 = 0.

<span class='text_page_counter'>(17)</span> Ûx=-6 hoÆc x=3 Vậy phơng trình đã cho có hai nghiệm là x = -6 và x = 3 . Mét sè ph¬ng tr×nh d¹ng t¬ng tù ph¬ng tr×nh (*) cũng có thể giải đợc theo cách trên : Gi¶i ph¬ng tr×nh (4x + 1)(12x - 1)(3x + 2)(x + 1) = 4 (8). Chó ý :. c.3. VÝ dô 8. . t=−3 ¿ t=2 ¿ ¿ ¿ ¿ Gi¶i :(8)⇔ [ ( 12 x −1 ) ( x+1 ) ][ ( 4 x+1 )( 3 x+ 2 ) ] =4 ⇔ ( 12 x 2+11 x −1 ) ( 12 x2 +11 x +2 )=4 ¿ ¿ §Æt : t =12 x 2 +11 x ta cã : (8) ⇔(t-1)(t+ 2)=4 ⇔ t 2 +t − 6=0 ⇔. +) Víi t = -3 th× 12x2 + 11x + 3 = 0 , ph¬ng tr×nh nµy v« nghiÖm. +) Víi t = 2 th× 12x2 + 11x - 2 = 0 ⇔ x= −11 ± √ 217 24 Vậy phơng trình đã cho có hai nghiệm − 11+ √ 217 vµ x1 = x2 = 24. − 11 − √ 217 24. d.. Ph¬ng tr×nh cã d¹ng. : ax4 + bx3 + cx2 + dx + e = 0 , víi. e b2 = a d2. * §Æc biÖt khi. d.1. ¿ e=a b=± d ' ¿{ ¿. Ta cã ph¬ng tr×nh : ax4 + bx3 + cx2 ± bx + d + a = 0 , Phơng trình này thờng đợc gọi là phơngtrình bậc bốn có hệ số đối xứng . Thö trùc tiÕp víi x = 0 , nÕu x ¹ 0 chia c¶ hai vÕ cña. C¸ch gi¶i :. phơng trình cho x2 và đặt t=x+ d bx hai Èn t d.2 :. VÝ dô. , råi ®a vÒ ph¬ng tr×nh bËc.

<span class='text_page_counter'>(18)</span> . VÝ dô 9 : Gi¶i ph¬ng tr×nh :. 2x4 - 21x3 + 74x2 - 105x + 50x = 0 (9). .. Gi¶i : Với x = 0 không phải là nghiệm của phơng trình đã cho . Chia c¶ hai vÕ cña ph¬ng tr×nh (9) cho x2 ta cã : 1 1 (9) Û 2 x 2  21x  74  105  50 2 0 x x 25  5   Û 2  x 2  2   21 x    74 0 . x  x   5 25 §Æt t  x  tacã t 2  10 x 2  2 VËy x x 9  t  (9) Û 2(t  10)  21t  74 0 Û 2t  21t  54 0 Û 2   t 6 5 9 x+ = ⇔ x 2 5 x= 2 9 ¿ +) Víi t= th× 2 x=2 ¿ ¿ ¿ ¿ ¿ 5 x+ =6 ⇔ x x=1 ¿ +) Víi t = 6 th× x=5 ¿ ¿ ¿ ¿ ¿ 2. 2. Vậy phơng trình đã cho có bốn nghiệm : x =1 ; x = 2; x = 5 ; x= 5. . Ph¬ng tr×nh bËc ba. 2. 4. a.. Ph¬ng tr×nh bËc ba lµ ph¬ng tr×nh d¹ng :. ax3 + bx2 + cx + d = 0 (a ¹ 0). (1) Trong đó a , b , c là những số thực đã cho , x là ẩn số . b. : phơng trình bậc ba tổng quát đã đợc nhà toán học C¸c®an« t×m ra , tuy nhiªn v× c«ng thøc nghiÖm qu¸ phøc t¹p nªn kh«ng tr×nh bµy . Trong ch¬ng tr×nh THCS , ph¬ng tr×nh (1) thêng. C¸ch gi¶i.

<span class='text_page_counter'>(19)</span> đợc giải dựa vào nhẩm và đoán một nghiệm nào đó của phơng tr×nh .Ta chó ý r»ng nÕu a + b + c + d = 0 th× x = 1 lµ nghiÖm cña ph¬ng tr×nh (1) , cßn nÕu a - b + c - d = 0 th× x = -1 lµ mét nghiÖm của phơng trình (1) . từ đó suy ra các nghiệm còn lại . c.. VÝ dô  VÝ dô 10 : Gi¶i ph¬ng tr×nh 2x. 3. + 5x2 + 2x - 9 = 0. (10) . Gi¶i : V× 2 + 5 + 2 - 9 = 0 nªn x0 = 1 lµ mét nghiÖm cña ph¬ng tr×nh (10) .Ta ph©n tÝch vÕ tr¸i cña (10) thµnh tÝch các nhân tử , trong đó có một nhân tử x - 1 , ta có : (10) Û (2x3 - 2x2) + (7x2 -7x) + (9x - 9) = 0 Û 2x2(x - 1) +7x(x - 1) + 9(x - 1) = 0 Û (x - 1)(2x2 + 7x + 9) = 0 x=1 ¿ 2 x 2 +7 x+ 9=0 , V« nghiÖm Û ¿ ¿ ¿ ¿. II. Bµi tËp. VËy nghiÖm cña ph¬ng tr×nh (10) cã nghiÖm duy nhÊt x = 1 .. 1. Gi¶i c¸c ph¬ng tr×nh : a. x+ 1 + x +2 = x+ 3 + x+ 4 b.. c.. d.. e.. .. 2004 2003 2002 2001 x+ 1¿2 ¿ . ¿ 2 x x 2+ ¿ x +1¿ 2 ¿ ¿ . 1 1 − x2 ¿ 4 x−4¿ ¿ x 2 − 3 ¿2 ¿ x −1 ¿2 . ¿ ¿ 1 x 2 −3 ¿ 4 + ¿ ¿ ¿ ¿ x +1 2 x +1 x−2 2 . + =12 x −2 x−4 x−4. ( ). ( ).

<span class='text_page_counter'>(20)</span> 3. f. g. a⋅. x −1 ¿ ¿ ¿ x3 x3 + ¿. .. 9 x8 +84 x6 +126 x 4 +36 x 2 +1 9 a8 +84 a6 +126 a 4+ 36 a2 +1 + x ⋅ =0 x 8 +36 x 6 +126 x 4 +84 x 2 +9 a8 +36 a6 +126 a 4 +84 a2+ 9. 2. Gi¶i c¸c ph¬ng tr×nh : a. x4 - 4x3 + 3x2 + 8x -10 = 0. b. 2x4 + 3x3 - 16x2 + 3x + 2 = 0. c. x4 - 2x3 - 6x2 +16x - 8 = 0. d. x4 + x2 + 4x - 3 = 0 . e. x4 - 3x2 - 10x - 4 = 0 . f. x3 + 7x2 + 7x + 2 = 0 . g. x3 + x2 + x = − 1 . 3 h. (x + 1)(x + 3)(x + 5)(x + 7) = 9 . i. (1 + x)2 +(x + 2)3 + (x + 3)4 = 2 . j. (1 + x)4 = 2(1 + x4) . k. (x + 3)4 + (x + 5)4 = 2 . l. (x2 + 3x - 4)2 + 3(x2 + 3x - 4) = x + 4 . m. x4 + (x - 1)(x2 - 2x + 2) = 0 . n. 2000(2001 - 2000x2)2 = 2001 - x. 3. a. T×m a sao cho ph¬ng tr×nh sau cã nghiÖm duy nhÊt vµ nghiÖm đó dơng 3 ax −2 3a − a= . x−2 a−2 b. Tìm m để phơng trình (3m -1)x + m = 2x + 1 có nghiệm với mäi gi¸ trÞ cña m . c. Tìm m để phơng trình x −m = x −2 có nghiệm duy nhất . x+ 1 x −1 b. Tìm m để phơng trình (m2 - m)x = m - 1 có nghiệm duy nhÊt . c. Tìm a để phơng trình sau có nghiệm duy nhất : x 2 −( 3 a− 2) x +2 a2 − 5 a −3 =0 x 2 +5 x − 14. .. d. T×m tÊt c¶ sè nguyªn d¬ng p > 1 sao cho ph¬ng tr×nh sau cã nghiÖm duy nhÊt : x 3+ px 2+ p −1+ 1. (. 4. Gi¶i ph¬ng tr×nh :. p −1. ) x +1=0. ..

<span class='text_page_counter'>(21)</span> 1 1+¿ ⋅ ⋅. ⋅ 1 x. 1+ 1+. 1 1 1+ 1+ ❑ ❑. ❑ ⏟ =x n. §Þnh lý ViÐt I. KiÕn thøc c¬ b¶n 1. Định lý viét đối với phơng trình bậc hai. b.. a.. §Þnh nghÜa thuËn : NÕu ph¬ng tr×nh bËc hai ax c. =0. (a. ¹. cã b c x 1+ x 2=− vµ x1 x 2= . a. Định lý đả. 0). hai. nghiÖm. x1. ,. + bx + x2 th×. 2. a. b. o :Cho hai số a , b . Khi đó chúng là nghiệm của phơng trình x2 + Sx + P = 0 , trong đó S = a + b ; P = a b c. + Cho phép nhẩm nghiệm trong những trờng hợp đơn giản . + Cho phép tính giá trị các biểu thức đối xứng của các nghiÖm vµ dÊu cña c¸c nghiÖm mµ kh«ng cÇn gi¶i ph¬ng tr×nh . d.. ý nghĩa của định lý Viéet. . VÝ dô VÝ dô 1 : Cho c¸c ph¬ng tr×nh x. + px + 1 = 0 vµ x 2 + qx + 2 = 0 cã c¸c nghiÖm lÇn lît lµ a , b vµ c , d . Chøng minh r»ng : (b - a)(b - c) = pq - 6 Gi¶i : Theo hÖ thøc ViÐt ta cã : 2. ¿ a+ b=− p ab=1 vµ ¿ b+ c=-q bc ==2 ¿{ ¿. Nªn (b - a)(b - c) = b2 - (a + c)b + ac = [b2 + (a +c)b + ac] -2(a +c)b = (b + a)(b + c) - 2(ab + bc) = (-p)(-q) - 2( 1+ 2) = pq - 6 . VËy (b - a)(b - c) = pq - 6 ..

<span class='text_page_counter'>(22)</span> . VÝ dô 2 : Cho m ¹ 0 vµ ph¬ng tr×nh mx. + px + q = 0 (1) cã hai nghiÖn d¬ng x1 , x2 . Chøng minh r»ng : a. Ph¬ng tr×nh qx2+ px + m = 0 (2) còng cã hai nghiÖm d¬ng x3,x4 b. x1 + x2 + x3 + x4 ³ 4 . 2. Gi¶i :. a. Ph¬ng tr×nh (1) cã hai nghiÖm d¬ng x1 , x2 nªn ¿ Δ1 ≥ 0 S1 =x1 + x 2 >0 P1=x 1 x 2 >0 tøc lµ 2 ¿ Δ 1= p −4 mp≥ 0 −p S 1=x 1+ x2 = >0 m q P1=x 1 x 2= >0 m ¿{{ ¿. §Ó ý r»ng ph¬ng tr×nh (2) cã D2 = p2 - 4mp = D1 ³ 0 nªn ph¬ng tr×nh (2) cã hai nghiÖm x3 , x4 . MÆt kh¸c P2= m >0 nªn x3 vµ x4 cïng dÊu , v× q −p −p m S 2= = ⋅ >0 q m q. nªn x3 vµ x4 cïng dÊu d¬ng .. b. Ta cã : x1 + x2 + x3 + x4 ¿ − p + − p . m q áp dụng bất đẳng thức Cauchy cho hai số dơng −p −p + , ta cã m. q. | p| . − p − p − p − pm − p − pm + = + ≥2 ⋅ =2⋅ m q m mq m mq √ mq 2 2 MÆt kh¸c v× 4mq Δ 1=p − 4 mp≥ 0 nªn p ³ | p| 2m ¿ 4 q > 0 , suy ra 2⋅ ≥2 . q √ mq | p| V× thÕ − p + − p ≥2 ⋅ ≥ 4 , hay x 1+ x 2 + x 3+ x 4 ≥ 4 . m q √ mq. √(. )(. ).

<span class='text_page_counter'>(23)</span> ¿ −p −p = m q p2=4 mq ⇔ ¿ m=q p=−2 m ¿{ ¿. §¼ng thøc x¶y ra khi vµ chØ khi. 2.. .. Định lý Vié t đối với phơng trình bậc cao a.. §Þnh lý thuËn : NÕu ph¬ng tr×nh bËc n :. 0) .. anxn + an-1xn-1 + ...+a1x + a0 = 0 ,. (a n ¹. Cã n nghiÖm x1 , x2 , ... , xn (c¸c nghiÖm kh«ng nhÊt thiÕt ph©n biÖt ) th× ta cã hÖ thøc ViÐt sau : x 1 + x 2+ ⋅+ x 2=−. a n− 1 . an. x 1 x 2+ ⋅+ x 1 x n + x 2 x 3+ ⋅+ x 2 x n +⋅+ x n −1 x n= −1 ¿k. an −2 . an. a n −k an. ¿ ¿ x i x i ⋅⋅ x i =¿ ∑ ¿ 1. ⋅⋅⋅⋅⋅⋅⋅⋅⋅⋅⋅⋅⋅⋅⋅⋅ ¿ a −1 ¿ n 0 . a1 x 1 x 2 ⋅⋅ x n=¿. 2. k. 1 ≤i 1<i 2 <⋅< ik ≤ n. . Định lý đảo : Cho n số thực tuỳ ý. b đặt. a1 , a2 , ... , an ,. S 1=α 1+ α 2+⋅+ α n . S 2=α 1 α 2 +⋅+ α n −1 α n . ⋅⋅⋅⋅⋅⋅ S k= ∑ α i α i ⋅⋅α i . 1 ≤i1 <i 2<⋅<i k ≤n. 1. 2. k. ⋅⋅⋅⋅ Sn =α 1 α 2 ⋅⋅ α n .. Th× an 1 , a2n-1 , ... , an lµ c¸c nghiÖm cña ph¬ng tr×nh : x - S1x +S2xn-2 - + (-1)kSkxn - k + ...+ (-1)Sn = 0 . Chẳng hạn định lý Viét cho phơng trình bậc ba phát biểu nh sau : NÕu : x1 , x2 , x3 lµ nghiÖm cña ph¬ng tr×nh bËc ba :.

<span class='text_page_counter'>(24)</span> ax3 + bx2 + cx + d = 0 .. Th× :. VÝ dô VÝ dô 3 : Gäi x. b x 1 + x 2+ x 3 =− . a c x 1 x 2+ x 2 x3 + x 3 x1 = . a d x 1 x2 x 3=− . a. c. . , x2 , x3 lµ ba nghiÖm cña ph¬ng tr×nh : x3 + px + q = 0 . p , q Î R Chøng minh r»ng x13 + x23 + x33 = 3x1x2x3 .. Gi¶i :. 1. Theo hÖ thøc ViÐt cho ph¬ng tr×nh bËc ba ta cã x1 + x2 + x3 = 0 . V× x13 + x23 + x33 - 3x1x2x3 = (x1 + x2 + x3)( x12 + x22 + x32 - x1x2 - x2x3 -x3x1) . Nªn x13 + x23 + x33 - 3x1x2x3 = 0 . Suy ra x13 + x23 + x33 = 3x1x2x3 .  Gi¶i ph¬ng tr×nh x3 + px2 + qx + r = 0 , biÕt r»ng gi÷a c¸c nghiÖm x1 , x2 , x3 cña ph¬ng tr×nh cã mèi liªn hÖ x1 = x2 + x3 Theo hÖ thøc ViÐt cho ph¬ng tr×nh bËc ba th×. VÝ dô 4 :. Gi¶i :. x 1+ x2 + x 3=− p . x 1 x 2 + x 2 x 3+ x3 x 1=q . x 1 x 2 x 3=− r .. (1) (2) (3). V× x1 = x2 + x3 nªn tõ (1) suy ra x1 = − p 2. x 2+ x 3=−. nh vËy th×. p 2 2. Theo (2) th× x 2 x 3=q − x1 (x 2 + x 3)=q − p 4 hai nghiÖm cña ph¬ng tr×nh :. . V× vËy x2 , x3 lµ X2+. p p2 X +q − =0 2 4. . Chó ý r»ng ph¬ng tr×nh nµy cã nghiÖm khi vµ chØ khi : 2. Δ=. 2. 2. p p 5p 5 2 −4 q − = − 4 q≥ 0⇔q≤ p 4 4 4 16. (. ). . Ch¼ng h¹n khi p = 4 , q = 1 th× x 2 , x3 lµ hai nghiÖm cña pt : X2 + 2X - 3 = 0 ..

<span class='text_page_counter'>(25)</span> Nªn. . Bµi tËp. ¿ x2=1 x 3=− 3 hoÆc ¿ x 2=−3 x 3 =1 ¿{ ¿. , suy ra x1 = x2 + x3 = -2 .. II 1.Gi¶ sö x1 , x2 lµ c¸c nghiÖm cña ph¬ng tr×nh x2 + px - 1 = 0 víi p lµ sè nguyªn lÎ . Chøng minh r»ng : víi sè tù nhiªn n tuú ý , c¸c sè S n= x1n + x2n vµ Sn+1= x1n+1 + x2n+1 lµ nh÷ng sè nguyªn vµ sè nguyªn tè cïng nhau . 2. T×m tÊt c¶ c¸c sè tù nhiªn m , n sao cho c¸c nghiÖm cña ph¬ng tr×nh x2 - m(n + 1)x + m + n + 1 = 0 còng lµ sè tù nhiªn . 3. Cho x1 , x2 lµ c¸c nghiÖm cña ph¬ng tr×nh x2 - 6x + 1 = 0 . Chøng minh r»ng : víi sè tù nhiªn n tuú ý sè S n= x1n + x2n lµ sè nguyªn vµ kh«ng lµ béi cña 5 . 4. Chøng minh r»ng nÕu a1a2 ³ 2(b1 + b2) th× Ýt nhÊt mét trong hai ph¬ng tr×nh sau cã nghiÖm : x2 + a1x + b1 = 0 (1) vµ x2 + a2x + b2 = 0 (2) . 5. Chøng minh r»ng trong ba ph¬ng tr×nh sau ®©y cã Ýt nhÊt mét ph¬ng tr×nh cã nghiÖm : x2 + 2ax + bc = 0 (1) x2 + 2bx + ca = 0 (2) x2 + 2cx + ab = 0 (3) 6. Cho x1 , x2 lµ hai nghiÖm cña ph¬ng tr×nh x2 - ax + 1 = 0 . a. H·y tÝnh S7 = x17 + x27 theo a . b. T×m ®a thøc bËc 7 cã hÖ sè nguyªn nhËn. α=. 2 75 + 5 2. √ √ 7. lµ. nghiÖm . 7. Ch a , b lµ hai sè d¬ng . BiÕt r»ng ph¬ng tr×nh x3 - x2 + 3ax - b = 0 cã ba 3 nghiÖm (kh«ng nhÊt thiÕt ph©n biÖt ) . Chøng minh r»ng a3 +27 b≥ 28 .. b. 8. Cho a , b , c là độ dài ba cạnh của một tam giác , a > b ³ c . Xác định tất cả các giá trị x > 0 sao cho a + x , b + x , c + x , là độ dài ba cạnh của mét tam gi¸c vu«ng . 9. Chøng minh r»ng : nÕu ph¬ng tr×nh x2 + px + q = 0 cã mét nghiÖm gÊp k lÇn mét nghiÖm cña ph¬ng tr×nh x2 + mx + n = 0 th× c¸c hÖ sè m , n , p , q tho¶ m·n hÖ thøc (q - k2n)2 - k(p - km)(knp - qm) = 0 . 10. Chứng minh rằng điều kiện cần và đủ để phơng trình ax2 + bx + c = 0 (ac ạ 0) có hai nghiệm trong đó có một nghiệm gấp k lần nghiệm kia (k ¹ 1) lµ kb2= (k + 1)2ac. 11. Trong mét cÆp nghiÖm cña ph¬ng tr×nh x2 - yx2 - y + 8x + 7 = 0 , h·y t×m cÆp nghiÖm (x , y) mµ y cã gi¸ trÞ lín nhÊt . 12. Víi gi¸ trÞ nguyªn nµo cña a , b th× ph¬ng tr×nh x2 + ax - b = 0 cã hai nghiÖm ph©n biÖt x1 , x2 tho¶ m·n -2 < x1 < -1 vµ 1 < x2 < 2 ..

<span class='text_page_counter'>(26)</span> 13. Chøng minh r»ng nÕu a , b , c lµ nh÷ng sè nguyªn lÎ th× ph¬ng tr×nh ax2 +bx +c=0 kh«ng cã nghiÖm h÷u tû . 14. Cho p=abc lµ mét sè nguyªn tè cã ba ch÷ sè . Chøng minh r»ng ph¬ng tr×nh ax2 + bx + c = 0 kh«ng cã nghiÖm h÷u tû . 15. Cho ph¬ng tr×nh ax2 - bx + b = 0 (ab > 0) cã c¸c nghiÖm lµ x 1 , x2 . Chøng minh r»ng tån t¹i c¸c sè a1 , a2 Î { -1 , 1 } sao cho : x1 x b + α 1 2 +α 2 =0 x2 x1 a. √ √ √. .. 16. Cho ph¬ng tr×nh ax3 + bx2 + cx + d = 0 (a ¹ 0) cã ba nghiÖm d¬ng lµ x1 , x2 , x3 . Chøng minh r»ng : x1 + x + x3 7. 7 2. 7. b3 c2 81 a5. .. 17. Chøng minh r»ng víi mäi sè thùc a , ph¬ng tr×nh bËc ba x3 - x2 + 18ax 2a = 0 kh«ng thÓ cã ba nghiÖm d¬ng ph©n biÖt . 18 . Gi¶i ph¬ng tr×nh bËc ba x3 + px2 + qx + r = 0 biÕt r»ng gi÷a c¸c nghiÖm x1 , x2 ,x3 cña nã cã mèi liªn hÖ x12 = x2x3 .. Chuyên đề 2. Phơng trình chứa dấu giá trị tuyệt đối. I. KiÕn thøc c¬ b¶n 1. D¹ng c¬ b¶n. ¿. |F ( x)|=G( x)⇔ G(x)≥ 0 F( x )=G(x ) hoÆc ¿ G( x )≥ 0 F ( x )=−G(x ) ¿ |F (x)|=|G( x)|⇔ F ( x )=G( x) hoÆc F( x)=-G( x ) ¿{ ¿. Chú ý : khi giải phơng trình chứa dấu giá trị tuyệt đối , ngời ta cũng hay sö dông ph¬ng ph¸p chia kho¶ng vµ ph¬ng ph¸p hµm sè .. 2. Mét sè vÝ dô :  VÝ dô 1 : Gi¶i ph¬ng tr×nh. |2 x −1|+|2 x +1|=4 . (1). Gi¶i :.  NÕu x ≥ 1 2. th× |2x-1|=2 x −1 ; |2x+ 1|=2 x+ 1 ..

<span class='text_page_counter'>(27)</span> (1) Û 2x - 1 + 2x + 1 = 4 Û 4x = 4 Û x = 1 , tho¶ m·n ®k  NÕu − 1 < x < 1 th× |2x-1|=1− 2 x ; |2x+ 1|=2 x+ 1 . 2 2 (1) Û 1 - 2x + 2x + 1 = 4 Û 0x = 2 , v«nghiÖm .  NÕu x ≤ − 1 th× |2x-1|=1− 2 x ; |2x+ 1|=− 2 x −1 . 2 (1) Û 1 - 2x - 2x - 1 = 4 Û -4x = 4 Û x= -1, tho¶ m·n ®k Vậy phơng trình đã cho có hai nghiệm là x = -1 và x = 1 .  Gi¶i ph¬ng tr×nh : |x 2+ x|=|5 x 2+7 x −1| (1) .. VÝ dô 2 :. ⇔ x + x=5 x +7 x − 1(1') ¿ 1 \) \{\} \} \} \[ \} \{ (1) ¿ 2 x + x == 5 x 2=7 x+ 1¿ ¿ ¿ ¿¿ (1')⇔ 4x 2+6 x −1=0 ⇔ −3+ √ 13 x= 4 ¿ −3 − √13 x= 4 ¿ ¿ ¿ ¿ ¿ 1 \) dlrarrow 6 x rSup \{ size 8\{2\} \} +8x - 1=0 dlrarrow alignl \{ stack \{\[x= 2. Gi¶i :. 2. VËy ph¬ng tr×nh (1) cã bèn nghiÖm : − 4 − √ 22 − 3− √ 13 − 4 + √ 22 −3+ √ 13 x 1= ; x2 = ; x 3= ; x 4= 6. . VÝ dô 3 : Gi¶i ph¬ng tr×nh. 4 6 2 |x − 5 x +5|=−2 x +10 x −11 (3) 2. Gi¶i :. (3)⇔ |x 2 −5 x +5|=−2( x 2 − 5 x+5) −1 . §Æt t = x2 - 5x + 5 ta cã : (3)⇔ |t|=− 2t − 1.. V× vÕ tr¸i kh«ng ©m nªn vÕ ph¶i còng kh«ng ©m , tøc lµ -2t - 1 ³ 0 hay t ≤ − 1 ⇒|t|=− t . 2. VËy (3) Û -t = -2t -1 Û t= -1 , tho¶ m·n ®iÒu kiÖn t ≤ − 1 . 2. 4.

<span class='text_page_counter'>(28)</span> 2. Do đó. 2. x −5 x +5=− 1 ⇔ x −5 x +6=0 ⇔ x=2 ¿ x=3 ¿ ¿ ¿ ¿ ¿. VËy ph¬ng tr×nh cã hai nghiÖm x = 2 vµ x = 3 .. II. Bµi tËp :. 3. Gi¶i ph¬ng tr×nh : a.. | x2 −5 x +4|=x + 4 . b.. |x-1x +1|=1 .. 3 =|x +3|. |x-4|−1 d . |x-1|− 2|x −2|+3|x −3|=4 . 2 3 e . |x-3| +| x − 4| =1. |3-2x|−|x| f. =5 . |3+2 x|+ x −2 g . x 2 − 2 x +8 x=|x 2 − 1|. k . x 2 − 5| x −1|−1=0 . l. |1-x|=1+ x+ x 2 . c.. 4. Gi¶i ph¬ng tr×nh : a . |3x-1|−|2 x +3|=0 . b . |2-3x 2|−|6 − x 2|=0 . c . 2|x|−| x −3|=3 . d . |x 2 −1|=−| x|+ 1. e . |7-2x|=|5 −3 x|+|x +2|. 2 x −1 f. =x . |x −2| x2 −1+|x +1| g. =2 . |x|( x − 2). 5. Víi gi¸ trÞ nµo cña tham sè a , ph¬ng tr×nh sau cã nghiÖm duy nhÊt. a . |2 x − a|+1=|x +3|. 1 b . |x-1|+|2 x+1|= . a. 6. BiÖn luËn theo tham sè m sè nghiÖm cña ph¬ng tr×nh |x − 1|+|x +2|=m ..

<span class='text_page_counter'>(29)</span> Chuyên đề 3. Ph¬ng tr×nh v« tû. I. KiÕn thøc c¬ b¶n 1. Ph¬ng tr×nh v« tû c¬ b¶n 2k. √ f (x)=g(x) ⇔. .  . g ( x)≥ 0. 2k. f ( x )=[ g( x) ] ¿{ 2k √ f (x )=2√k g( x)⇔ f (x) ≥0 (hoÆc g (x) ≥ 0) f (x)=g (x) ¿{ 2 k+1 2 k+1 √ f ( x )=g( x )⇔ f ( x )=[ g( x )]. (Trong c¸c biÓu thøc trªn , k lµ sè nguyªn d¬ng ) Nếu gặp các dạng không cơ bản , phải đặt điều kiện cho căn thức bậc chẵn có nghĩa , rồi biến đổi để đa về dạng cơ b¶n .. Chó ý :. 2. Mét sè vÝ dô  VÝ dô 1: Gi¶i ph¬ng tr×nh : √ 2 x +1+ √ x −3=4 (1) . Giải : Điều kiện để các căn thức có nghĩa x ³ 3 (*) .. (1) ⇔ 2 x+ 1+ x −3+2 √(2 x +1)( x −3)=16 ⇔ 2 √ (2 x +1)( x −3)=3 (x − 6) Vế phải không âm vì vế trái không âm do đó x Ê 0 (**) . a. Û x2 - 88x + 336 = 0 ⇔ x=4 tho¶ m·n (∗)vµ (**) ¿ x=84 kh«ng tho¶ m·n(∗)vµ (**) ¿ ¿ ¿ ¿ ¿. VËy ph¬ng tr×nh cã nghiÖm duy lµ x = 4 . NÕu ph¬ng tr×nh cã chøa c¨n thøc bËc ch½n th× ta chØ đợc nâng lên luỹ thừa bậc chẵn hai vế của phơng trình khi hai cña ph¬ng tr×nh kh«ng ©m .. Chó ý:.

<span class='text_page_counter'>(30)</span> VÝ dô 2 : Gi¶i ph¬ng tr×nh. . √ 2 x 2 +8 x+ 6+√ x2 −1=2 x +2. (2) .. Gi¶i :. Điều kiện để các căn thức ở phơng trình (2) có nghĩa x ³ -1 (*) . (2). ( x −1) x +1 ⇔ √ 2( x +1)( x +3)+ √ ¿ ¿=2( x+1). (2’) ..  x = -1 lµ nghiÖm cña ph¬ng tr×nh (2’) .  NÕu x ¹ -1 tøc lµ x + 1 > 0 th× : 2 \) . \} \{. (2’). ¿ ¿ ⇔ √ 2( x +3)+ √ x −1=2 √ x+ 1 ¿. Điều kiện để các căn thức ở phơng trình (2”) có nghĩa x ³ 1 (**). (2 \) dlrarrow 2 \( x+1 \) +x - 1+2 sqrt \{2 \( x+3 \) \( x - 1 \) \} =4 \( x+1 \) \} \{.  x = 1 lµ nghiÖm cña ph¬ng tr×nh (2”’) .  NÕu x ¹ 1 tøc lµ x > 1 th× :. 2 ' \) dlrarrow 2 sqrt \{2 \( x+3 \) \} = sqrt \{x - 1\} dlrarrow 8 \( x+3 \) =x - 1\} \{\} #. Vậy phơng trình đã cho có hai nghiệm x = -1 và x = 1 . NhiÒu ph¬ng tr×nh v« tû cã thÓ gi¶i rÊt ng¾n gän b»ng phơng pháp đánh giá : Gi¶i ph¬ng tr×nh √3 x+1+ √3 x +2+ √3 x +3=0. Chó ý : . VÝ dô 3. Gi¶i :.  Ta cã x = -2 lµ mét nghiÖm cña ph¬ng tr×nh . ¿ x+1<− 1 √ 3  NÕu x < -2 th× √3 x +2<0 nªn √ x+3< 1 ¿{{ ¿ 3 3 3 x+1+ x +2+ x +3< 0 . √ √ √ ¿ 3 √ x+1>− 1 3  NÕu x > -2 th× √3 x +2>0 nªn √ x+3> 1 ¿{{ ¿ 3 3 3 √ x+1+ √ x +2+ √ x +3> 0 . 3. Vậy phơng trình đã cho có một nghiệm duy nhất là x = -2 ..

<span class='text_page_counter'>(31)</span> Chó ý : NhiÒu khi ph¬ng ph¸p nh©n liªn hîp tá ra rÊt hiÖu qu¶ khi gi¶i c¸c ph¬ng tr×nh v« tû .. VÝ dô 4 : Gi¶i ph¬ng tr×nh : √ 4 x +1− √ 3 x −2= x +3 (4) . 5 Giải : Điều kiện để các căn thức ở phơng trình (4) có nghĩa x. . ³ 2 . 3. (4 x +1)−(3 x − 2) x+3 x+3 x+ 3 = ⇔ = 5 √ 4 x +1+√ 3 x −2 √ 4 x +1+ √ 3 x −2 5 ⇔ √ 4 x +1+ √ 3 x −2=5(4') 2 thì hàm số f(x) = √ 4 x +1+ √ 3 x −2 đồng biến và 3. (4 )⇔. Khi x ³ f(2) =5 nªn ph¬ng tr×nh (4’) cã nghiÖm duy nhÊt lµ x = 2 , ®©y cũng là nghiệm duy nhất của phơng trình đã cho . có thể giải phơng trình vô tỷ bằng cách đặt ẩn phụ và chuyển về phơng trình đại số hoặc hệ phơng trình đại số đơn giản và dễ giải hơn .  Gi¶i ph¬ng tr×nh : √4 97 − x + √4 x −15=4 (5) .. Chó ý :. VÝ dô 5 :. Gi¶i :. §Æt u= 4√97 − x , v=√4 x − 15 th× u ³ 0 , v ³ 0 , u4 + v4 = 82 . (5)⇔ u, v≥0 u+ v=4 (1) 4 u + v 4 =82 (2) ¿{{. Tõ (1) Þv = 4 - u,thay v = 4 - u vµo (2) ta cã : u4+(4 - u)4=82 (2’) §Æt t = u - 2 ta cã : (2’) Û (t + 2)4 + (t - 2)4 = 82 Û t4 + 24t2 - 25 = 0 Û t2 = 1.

<span class='text_page_counter'>(32)</span> Û. ¿ u=1 v=3 ¿ t=1 ⇒ ¿ ¿ u=3 ¿ v=1 ¿ ¿ ¿ ¿ ⇒ ¿ t=−1 ⇒ ¿. Vậy phơng trình đã cho có hai nghiệm x = 16 và x = 96 .. II. Bµi tËp. 1. đây là đúng :. 2.. Với giá trị nào của x thì mỗi đẳng thức sau a. b. c.. √ x + √ 2x-1+ √ x − √2 x − 1=√ 2 √ x + √ 2x-1+ √ x − √2 x − 1=1 √ x + √ 2x-1+ √ x − √2 x −1=2. Gi¶i c¸c ph¬ng tr×nh sau :. a. 8 x 2  b.. 4. 1 5  x 2. 3 x   2x 8. c. x 2  2 x  4 3 x 3  4 x d.. x(x  1)  x( x  2)  x( x  3 1 1 e. + x √ 2 − x2 3 f . 2x 2 −11 x +21=3 √ 4 x − 4 3 3 2 g . x −3 x − 8 x + 40=8 √ 4 x+ 4. √. √. k. ⏟ x +2 x +2 √ x +⋅+ 2 √ x +2 √ 3x =x. 3.. n. Gi¶i c¸c ph¬ng tr×nh sau :.

<span class='text_page_counter'>(33)</span> a.. x  2 x-1  x  2 x-1  x  1. b.. x  3  4 x-1  x  8  6 x-1 1. c. ( x 2  1  x)5  ( x 2  1  x)5 123 d.. x-1  x  3  2 ( x  1)( x  3) 4  2 x. e.. x. x 1 . x  4  x  9 0 x  1 3 x  2 2 x 2  5 x  3  16. f.. 2x  3 . g.. x 3  x2  1 . x 3  x 2  2 3. 1- x 4  x 2  x  1. h.. 1 z  2004  ( x  y  z ) 2 2 l. √ x-94+ √96 − x=x −190 x +9027 m. x+ y+ z + 4=2 √ x-2+4 √ y −3+ 6 √ z −5 2 2 2 2 n . √3x −7 x +3 − √ x − 2=√ 3x −5 x − 1− √ 3x − 3 x+ 4 2 p. √ x +1+ 2( x+ 1)=( x − 1) √ 1− x+3 √ 1− x. k.. x-2 . y  2003 . 3. q . ( √ x-1+1 ) +2 √ x −1=2 − x. 4. Gi¶i vµ biÖn luËn ph¬ng tr×nh :. a . √ a- √ a+ x=x , a lµ tham sè . b . ( x − 5 x+6) √ x2 −5 ax +6 a2 , a lµ tham sè . 1 1 1 c . x2 +2 ax+ =− a+ a 2+ x − , a lµ tham sè vµ 0< a< . 16 16 4 2. √ √ √√. 3 x+1+ x + + x=a , a lµ tham sè . 4 n n a+ x a+ x b √n x e. √ + = , a , b , c lµ tham sè . a x c f . √ x2 +2 px − p2 − √ p 2 −2 px − p2=1 , p lµ tham sè vµ p > 0 . d..

<span class='text_page_counter'>(34)</span>

<span class='text_page_counter'>(35)</span>

<span class='text_page_counter'>(36)</span>

<span class='text_page_counter'>(37)</span>

<span class='text_page_counter'>(38)</span>

<span class='text_page_counter'>(39)</span>

<span class='text_page_counter'>(40)</span>

<span class='text_page_counter'>(41)</span>

<span class='text_page_counter'>(42)</span>

<span class='text_page_counter'>(43)</span>

<span class='text_page_counter'>(44)</span>

<span class='text_page_counter'>(45)</span>

<span class='text_page_counter'>(46)</span>

<span class='text_page_counter'>(47)</span>

<span class='text_page_counter'>(48)</span>

<span class='text_page_counter'>(49)</span>

<span class='text_page_counter'>(50)</span>

<span class='text_page_counter'>(51)</span>

<span class='text_page_counter'>(52)</span>

<span class='text_page_counter'>(53)</span>

<span class='text_page_counter'>(54)</span>

<span class='text_page_counter'>(55)</span>

<span class='text_page_counter'>(56)</span>

<span class='text_page_counter'>(57)</span>

<span class='text_page_counter'>(58)</span>

<span class='text_page_counter'>(59)</span>

<span class='text_page_counter'>(60)</span>

<span class='text_page_counter'>(61)</span>

<span class='text_page_counter'>(62)</span>

<span class='text_page_counter'>(63)</span> HÖ Ph¬ng tr×nh Chuyên đề 1. HÖ Ph¬ng tr×nh h÷u tû. III. KiÕn thøc c¬ b¶n 7. Hệ phơng trình đối xứng loại một a.. Định nghĩa : Một hệ phơng trình hai ẩn x , y đợc gọi là. hệ phơng trình đối xứng loại một nếu mỗi phơng trình của hệ đã cho đối xứng với hai ẩn x , y (nghĩa là mỗi phơng trình của hệ không thay đổi khi ta đổi vai trò của x , y cho nhau ) . b. TÝnh chÊt : NÕu (x0 , y0) lµ mét nghiÖm cña hÖ th× (y 0 , x0) còng lµ nghiÖm cña hÖ . c. §Æt S = x+y , P = xy , víi S2 ³ 4P .. C¸ch gi¶i thêng dïng :. d.. VÝ dô 1 : Gi¶i hÖ ph¬ng tr×nh : Gi¶i : §Æt. ¿ S=x + y P=xy ¿{ ¿. ta cã :. ¿ x+ y+ xy=11 x 2+ y 2 +3( x+ y )=28 ¿{ ¿. (I)..

<span class='text_page_counter'>(64)</span> ¿ (I )⇔ S+ P=11 S2 −2 P+ 3 S=28 ⇔ ¿ P=11 − S 2 S − 2(11 − S)+3 S=28 ¿ ⇔ ¿ P=11 − S 2 S +5 S −50=0 ⇔ ¿ P=11 − S S=5 hoÆc S=-10 ¿ { ¿.  NÕu S = 5 th× P = 6 nªn x , y lµ c¸ nghiÖm cña ph¬ng tr×nh : 2. t −5 t +6=0 ⇔ t=2 ¿ t=3 ¿ ( x ; y )=(2 ; 3) ¿ (x ; y )=(3 ; 2) ¿ ¿ ¿ ⇒¿ ¿ ¿ ¿.  NÕu S = -10 th× P = 21 nªn x , y lµ c¸ nghiÖm cña ph¬ng tr×nh : 2. t +10 t +21=0 ⇔ t=−7 ¿ t=−3 ¿ ( x , y )=(−7 ; -3) ¿ ( x , y )=(−3 ; -7) ¿ ¿ ¿ ⇒¿ ¿ ¿ ¿. VËy hÖ ph¬ng tr×nh cã bèn nghiÖm :.

<span class='text_page_counter'>(65)</span> (x ; y) Î {(2 ; 3) ; (3 ; 2) ; (-7 ; -3) ; (-3 ; -7)}.. 8. Hệ phơng trình đối xứng loại hai a.. Định nghĩa : Một hệ phơng trình hai ẩn x , y đợc gọi là. hệ phơng trình đối xứng loại hai nếu trong hệ phơng trình khi ta đổi vai trò của x và y cho nhau thì phơng trình này trë thµnh ph¬ng tr×nh kia . b. TÝnh ChÊt : NÕu (x0 , y0) lµ mét nghiÖm cña hÖ th× (y 0 , x0) còng lµ nghiÖm cña hÖ . c. : trừ các vế tơng ứng của hai phơng trình thì nhận đợc phơng trình tích dạng : (x - y)f(x , y) = 0 hoÆc f(x , y) = 0 .. C¸ch gi¶i thêng dïng. d. VÝ dô 2 : gi¶i hÖ ph¬ng tr×nh. ¿ 3 x +1=2 y 3 y +1=2 x ¿{ ¿. (1) (2). (II) .. Gi¶i : Trõ tõng vÕ ph¬ng tr×nh (1) cho ph¬ng tr×nh (2) ta cã : x3 - y3 = 2(y - x) Û (x - y)(x2 + xy + y2 + 2) = 0 Ûx-y=0Ûx=y ( v× x2 + xy + y2 + 2 =. y 2 3 2 + y +2>0 2 4. ( ) x+. víi mäi gi¸ trÞ x , y ) .. Thay y = x vµo ph¬ng tr×nh (1) ta cã :. x3 - 2x + 1 = 0 Û (x - 1)(x2 + x - 1) = 0 Û. x =1 ¿ −1+ √ 5 x= 2 ¿ −1− √ 5 x= 2 ¿ ¿ ¿ ¿. Vậy hệ phơng trình đã cho có ba nghiệm : (x ; y) (1 ; 1 ) ; −1 − √ 5 ; −1 − √ 5 ; −1+ √ 5 ; −1+ √5. { (. 2. 9. Hệ phơng trình đẳng cấp :. 2. )(. 2. 2. .. )}. §a thøc hai biÕn x , y cã d¹ng : P(x , y) = anxn + an-1xn-1y + an-2xn-2y2 + ... + a2x2yn-2 + a1xyn-1 +a0yn-1 , trong đó nlà số tự nhiên , a0 , a1 , ... , an là những số thực không đồng thời bằng không , đợc gọi là đa thức đẳng cấp bậc n . a. Định nghĩa : Hệ phơng trình đẳng cấp là hệ có dạng.

<span class='text_page_counter'>(66)</span> ¿ f 1(x , y)=g 1 ( x , y ) g2 (x , y )=f 2 ( x , y ) ¿{ ¿. Trong đó f1(x , y) và f2(x , y) là hai đa thức đẳng cấp cùng bậc ; g1(x,y) và g2(x , y) là hai đa thức đẳng cấp cùng bậc . Ta thờng gặp hệ phơng trình vế trái đẳng cấp bậc hai dạng : ¿ a1 x 2+ b1 xy+ c 1 y 2=d1 a2 x 2+ b2 xy+ c 2 y 2=d2 ¿{ ¿. và hệ phơng trình vế trái đẳng cấp bậc ba dạng :. 10.. ¿ 2 2 2 3 a1 x + b1 x y+ c 1 xy +d 1 y =e 1 2 2 2 3 a2 x + b2 x y+ c 2 xy +d 2 y =e 2 ¿{ ¿. C¸ch gi¶i thêng gÆp :  Trêng hîp y = 0 ta xÐt trùc tiÕp .  Nếu y ạ 0 thì ta đặt ẩn phụ t= x ta nhận đợc một y hệ phơng trình mới (ẩn t và y ) tơng đối đơn giản hơn vµ dÔ gi¶i h¬n .  Ta khö y vµ gi¶i ph¬ng tr×nh Èn t , råi t×m y vµ x theo t đã biết . Chú ý : Có thể xét trực tiếp x = 0 , nếu x ạ 0 thì đặt t = b.. VÝ. y x. dô. vµ gi¶i t¬ng tù c¸ch trªn .. 3. ¿ 2 2 x −2 xy+3 y =9 2 2 2x −13 xy+15 y =0 ¿{ ¿. Gi¶i :. Gi¶i. hÖ. ph¬ng. tr×nh. :. (1) (2).  NÕu y = 0 th× tõ (1) suy ra x 2 = 9 , cßn tõ (2) suy ra x2 = 0 , vËy trong trêng hîp nµy v« nghiÖm .  Nếu y ạ 0 đặt x = ty , thay vào phơng trình (2) ta cã :.

<span class='text_page_counter'>(67)</span> y2(2t2 - 13t + 15) = 0 Û 2t2 - 13t + 15 = 0. ⇔ t=5 ¿ 3 t= 2 ¿ ¿ ¿ ¿ ¿. o NÕu t = 5 th× x = 5y thay vµo ph¬ng tr×nh (1) ta cã : 1 2 y= ⇔ 2 2 y= √ 2 ¿ √ 2 khi đó y=− 2 ¿. 3. ¿ ¿ ¿. o NÕu t = th× x = 2 tr×nh (1) ta cã :. 5 √2 2 ¿ 5 2 x=− √ 2 ¿ ¿ ¿ ¿ x=. 3 y , thay vµo ph¬ng 2. y 2=4 ⇔ y=2 ¿ y =−2 khi đó ¿ ¿ ¿ ¿. x=3 ¿ x=−3 ¿ ¿ ¿ ¿. Vậy hệ phơng trình đã cho có bốn nghiệm : 5 √2 √ 2 5 2 2 (x ; y) ; ; − √ ; − √ ; ( 3; 2 ) ; ( −3 ; − 2 ). {(. 2. 2. )(. 2. 2. ). .. 11.. Mét sè hÖ ph¬ng tr×nh d¹ng kh¸c a.. VÝ dô 4 : Gi¶i hÖ ph¬ng tr×nh theo tham sè a , b , c , d : ¿ y + z +u=a x + z+ u=b x + y +u=c x+ y+ z=d ¿{{{ ¿. }.

<span class='text_page_counter'>(68)</span> Gi¶i : Céng vÕ theo vÕ cña c¶ bèn ph¬ng tr×nh ta cã :. x + y + z + u = 1 (a + b + c + d) . 3 LÊy ph¬ng tr×nh nµy trõ vÕ theo vÕ cho ph¬ng tr×nh thø nhÊt x = 1 (-2a + b + c + d) . 3 Tơng tự ta tính đợc y = 1 (a -2 b + c + d) . 3 z = 1 (a + b - 2c + d) . 3 u = 1 (a + b + c - 2d) . 3. b.. VÝ dô 5 : Gi¶i hÖ ph¬ng tr×nh. ¿ ax+ by +cz=a+b+ c bx+cy +az=a+b+ c cx+ ay+ bz=a+b+ c ¿{ { ¿. Với a , b , c không đồng thời bằng không và a + b + c ạ 0 . Giải : Đặt X = x - 1 , Y = y - 1 , Z = z - 1 thì hệ đã cho trở thành : ¿ aX+ bY+cZ=0 bX+ cY+aZ=0 cX+aY+ bZ=0 ¿ {{ ¿. Céng vÕ theo vÕ c¶ ba ph¬ng tr×nh , råi chia cho a + b + c ¹ 0 ta cã : X+Y+Z=0. Thay Z = -(X + Y) vµo hai ph¬ng tr×nh ®Çu cña hÖ trªn ta cã : ¿ (a − c) X +(b −c )Y =0 (b − a) X +(c −a)Y =0 ¿{ ¿. Nh©n hai vÕ cña ph¬ng tr×nh ®Çu víi (a - c) vµ hai vÕ cña ph¬ng tr×nh thø hai víi (b - c) , råi céng vÕ theo vÕ ta cã : [( a - c)2 + (b - a)(b - c)] X = 0 (1). Mµ : ( a - c)2 + (b - a)(b - c) = a2 + b2 + c2 - ab - ac - bc = 1 [ ( a −b )2+ ( b −c )2 + ( c − a )2 ] >0 . 2. Nªn (1) Û X = 0 . Tính toán tơng tự thì Y = 0 , suy ra Z = 0 . Vậy hệ phơng trình đã cho cã nghiÖm duy nhÊt : x = y = z = 1 ..

<span class='text_page_counter'>(69)</span> IV. Bµi tËp 12.. Gi¶i c¸c hÖ ph¬ng tr×nh sau : ¿ ¿ b ¿(b+c y++a) z)-ax=b-c¿ +a)(z + x) -by=c-a a( c+ a) y +(a+b) z- (b+ c) x=2a 3 ¿ (a+b) z+(b+ c) x)( −(c y=2 b3 ¿((c b+c )x +(c+ a) y −(a+¿. biÕt a + b ¹ 0 , b + c ¹ 0 , c + a ¹ biÕt a + b + c ¹ 0 . 0. Cho (x , y) lµ nghiÖm cña hÖ ph¬ng tr×nh :. 13.. ¿ x+ y=2 a −1 x 2+ y 2 =a2 +2 a −3 ¿{ ¿. Xác định a để tích xy đạt giá trị nhỏ nhất , tìm giá trị nhỏ nhất đó . Cho hÖ ph¬ng tr×nh :. 14.. ¿ x+ y+ xy=m+1 x 2 y + xy 2=m ¿{ ¿. a) Gi¶i hÖ víi m = -2 . b) Tìm tất cả các giá trị của m để hệ có nghiệm (x , y) với x < 0 vµ y < 0 . Gi¶i hÖ : xi2 = 2002xi+1 + 1 , víi mäi i = 1 , 2 , 3 ... , 2002(Qui íc x2003=x1) T×m bèn sè thùc x1 , x2 , x3 , x4 sao cho mçi sè céng víi tích các số còn lại đều bằng 2 . Gi¶i c¸c hÖ ph¬ng tr×nh sau :. 15.. 16. 17..

<span class='text_page_counter'>(70)</span>  2 1 x  x  y 2  y 7  2 x  y  1  b)   3  2 y  x  1  x 2  1 3  y2  x 2  y 2 10  xy  x  y 19 d)  2  2  x  y 4  x y  xy 84  x 2  xy  y 2 4  x 2  y 2 65 g)    x  xy  y 2 ( x  1)( y  1) 18  x 2  3 y  x  x 3 7 x  3 y i)  3  2  y  3 x  y  y 7 y  3 x 2 2 2 2 3x  2 xy  y 11 2y(x  y ) 3x l)  2  2 2 2  x  2 xy  3y 17  x( x  y ) 10 y 3. a). c) f) h) k). 18.. Cho a , b , c là độ dài ba cạnh của một tam giác . Giải hệ ph¬ng tr×nh Èn x y , z sau : 3. 3. y + z =2 a( xy +yz +zx) 3 3 z + x =2b (xy + yz+zx ) x 3+ y3 =2 c (xy + yz+zx ) ¿{{. 19.. Cho hÖ ph¬ng tr×nh Èn x1 , x2 , ... , xn ¿ ax 1 + bx 1+ c=x 2 ax 2 + bx 2+ c=x 3 ¿ ax n −1 + bx n −1 +c =xn ax n + bx n +c=x 1 ¿ {{ { { ¿ 2. 2. 2. 2. Trong đó a , b , c là những số thực và a ạ 0 , chứng minh rằng : a) HÖ kh«ng cã nghiÖm thùc nÕu (b - 1)2 - 4ac < 0 . b) HÖ cã mét nghiÖm duy nhÊt nÕu (b - 1)2 - 4ac = 0 . c) HÖ cã h¬n mét nghiÖm thùc nÕu (b - 1)2 - 4ac > 0 .. 20.. a. Cho hÖ ph¬ng tr×nh :. b. Cho ba số a , b , c đôi một kh¸c nhau , (x , y , z) lµ mét nghiÖm cña hÖ ph¬ng tr×nh :.

<span class='text_page_counter'>(71)</span> ¿ x+ y+ z=3 3 3 3 x + y + z =15 x 4 + y 4 + z 4 =35 ¿{{ ¿. ¿ ( x − a)( y − a)( z − a)=d ( x − b)( y − b)( z − b)=d ( x − c)( y −c )( z − c)=d ¿{{ ¿. Cã mét nghiÖm (x , y , z) tho¶ H·y tÝnh : P = x3 + y3 + z3 2 2 2 m·n ®iÒu kiªn x + y + z < 10 H·y tÝnh : S = x5 + y5 + z5 Gi¶i c¸c hÖ ph¬ng tr×nh sau :. 21.. y − 3 x ¿ xy + xz+ xt+yz x +y a ¿ x+ y+ z=0 ¿ x 2+ y 2 + z 2=10 ¿ x 7+ y 7+ z7 =350 ¿ ¿x¿+3y b ¿ =3 ¿ ¿ x+ h ¿ x+ ¿ y y−+ z2+t=12 =0¿ ¿ ¿ i ¿ ¿ ¿ + 2 2 2 5 4 2 a-q b x +y x +y ¿ x − y +2 x y=2 4. y 5 − z 4 +2 y 2 z=2 ¿ 1x+ y+ z=3 ¿ x2 + y 2 + z 2=3 ¿ x 5+ y5 + z 5=3 ¿ ¿ { { ¿ z 5 − x 4 +2 z2 x=2 4 4 k ¿ − 2 y = y −2 z=z −2 x=− ¿ f ¿ ¿{ { ¿ ¿ 2 ¿ e x  x 2  4 xy  y 2 4  x 3  y 3 7 m)  n)  2  xy ( x  y ) 2  y  3xy 4  x  3xy  y  1 p)  2 2 3 x  xy  3 y 13.   1  (x  y)  1   5   xy  q)   x 2  y 2  1  1  9   x2 y2     . y   x-3y 4 x r)   y  3 x 4 x y . 1 3  2x  y  x  s)  2y  1  3 x y .  x 3 3 x  8 y t)  3  y 3 y  8 x. 5x 3  3 y 3 6  2 xy v)  3 3 3 x  2 y 8  3 xy. 2. 2. ¿ 2 l ¿|za+ − a x +|2ax1 −a a+1 −y a)=4 =1¿ | g ¿(x + y)( y+ z)=4xy2 z ¿ ( y + z)( x )=4 ¿ ( z+ y |¿a¿2¿−¿a¿{ { 1¿+¿|a2 −a3| x 3 1 2 2yz 3| xx)( 3 +|x 4| x 4zx 1| x. 22.. ( a1 , a2 , a3 , a4 lµ bèn sè kh¸c nhau cho tríc ) Gi¶i c¸c hÖ ph¬ng tr×nh sau :.

<span class='text_page_counter'>(72)</span>  x 2 2 x  y  2  y 2 y  z a)  2  z 2 z  t t 2 2t  x  2x1  5 x2  3x3 0 2x  5 x  3x 0  3 4 b)  2 ... 2x n  5 x1  3x2 0  x 3  y 3  x 2 ( y  z )  xyz  14  c)  y 3  z 3  y 2 ( z  x)  xyz  21 z 3  x 3  z 2 ( x  y )  xyz  7   x 3 ( y 2  3 y  3) 3 y 2  d )  y3 ( z 2  3 z  3) 3 z 2 z 3 ( x 2  3 x  3) 3x 2   x 3  9 y 2  27 y  27 0  e)  y3  9 z 2  27 z  27 0  z3  9 x 2  27 x  27 0  3-x ¿ 3.  2x 2  x2 1  y   3 y3 k)  4 z 2 y  y  1   4z3 x  6  z  z 4  z 2 1 1  2x1  x2  x 2  1  2x 2  x3  x 3  l ) ...  1 2x 2001  x2002  x2002   1 2x 2002  x1  x1   x(x  y  z) a-yz  m)  y(x  y  z) b-zx ,(a , b , c  0 z(x  y  z) c-xy  ¿. ¿ (2 z − y )( y +2)=9+ 4 y ¿ x 2 + z 2=4 x ¿ z≥0 f ¿ 2y 3 +2 x 2 +3 x+3=0 ¿ 2z 3+2 y 2 +3 y +3=0¿ 2x 3 +2 z 2 +3 z +3=0¿ ¿ ¿ g ¿ ¿ ¿ y +2=¿ ¿ 2003 2003 2003 2003 n ¿ x1 + x 2 − =x 3 ¿ x 2+ x 3 − =x 4 ¿. . .¿ x 2002 + x 2003 − =x 1 ¿ x 2003 + x 1 − = x1 x2 x2 x3 x 2002 x 2003 x2003 x 1 ¿ ¿ 1 1 1 1 1 1 1 1 1 x+3 ¿3 =3 −2 y l¿ + = ¿ + = ¿ + = ¿ ¿{ { ¿ ¿ x+ y z 5 z + x y 8 y+ z x 9 z 2+ 4 y 2=8 y ¿ (2 z − x )(x +3)=5 x+16 h ¿(¿ z ≥ 0¿¿¿i) ¿ ¿ x + y + z=1 ¿ x 4 + y 4 + z 4 =xyz ¿ ¿¿ { { { ¿ ¿. Chuyên đề 2.

<span class='text_page_counter'>(73)</span> HÖ Ph¬ng tr×nh v« tû. V. KiÕn thøc c¬ b¶n 23.. VÝ dô 1: Gi¶i hÖ ph¬ng tr×nh: ¿. √ x+1+ √ 7 − y =4 √ y+ 1+ √ 7 − x=4 ¿{ ¿. Gi¶i: §iÒu kiÖn -1 £ x £ 7 , -1 £ y £ 7 .. Tõ hÖ ph¬ng tr×nh suy ra √ x+1+ √ 7 − y =√ y+1+ √ 7 − x ⇔ √ x+ 1− √7 − x=√ y+ 1− √ 7 − y (1) . Ta có hàm số f(x) = √ x+1 − √ 7 − x đồng biến trên đoạn [-1 ; 7] Nªn tõ (1) suy ra x = y . VËy hÖ ph¬ng tr×nh trë thµnh : √ x+1+ √ 7 − x = 4 Mặt khác áp dụng bất đẳng thức bunhiacôpxki ta có : ( 1⋅ √ x +1+1⋅ √ 7 − x )2 £ (12 + 12)(x + 1 + 7 - x) = 16 Û √ x+1+ √ 7 − x £ 4 . DÊu b»ng x¶y ra khi vµ chØ khi x + 1 = 7 - x Û x = 3 . Vậy hệ phơng trình đã cho có nghiệm duy nhất là ( x ; y) = (3 ; 3). ¿ x+ √ y −1=1 4 y + √ x −1=1 ¿{ ¿ 4. 24.. VÝ dô 2 : Gi¶i hÖ ph¬ng tr×nh :. Gi¶i : §Ó c¸c c¨n thøc cã nghÜa th× x ³ 1 vµ y ³ 1 .. Khi đó x+ √4 y −1 ³ 1, dấu bằng xảy ra khi và chỉ khi x = y = 1 . thử lại ta có x = y = 1 là nghiệm của hệ đã cho .Vậy hệ phơng tr×nh cã nghiÖm duy nhÊt (x ; y) = (1 ; 1) . ¿. 25.. VÝ dô. √ x+1+ √ y −2= √ m 3 : cho hÖ ph¬ng tr×nh √ y+ 1+ √ x −2= √m ¿{ ¿. a) Tìm tất cả các giá trị của m để hệ có nghiệm . b) Gi¶i hÖ ph¬ng tr×nh khi m = 9 .. Gi¶i :.

<span class='text_page_counter'>(74)</span> §iÒu kiÖn. ¿ x≥2 y ≥2 m≥ 0 ¿{ { ¿. a) Từ hệ đã cho ta có : 2 2 √ x+1+ √ y −2= √ y +1+ √ x − 2⇔ ( √ x +1+ √ y − 2 ) =( √ y +1+√ x − 2 ) ⇔ √ (x +1)( y − 2)= √( y +1)(x −2) ⇔xy + y-2x-2=xy + x-2y-2 ⇔ x= y. Hệ phơng trình đã cho trở thành : √ x+1+ √ x − 2=√ m (1). Vì hàm số f(x) = √ x+1+ √ x − 2 đồng biến trên [2 ; +Ơ) nªn gi¸ trÞ nhá nhÊt cña hµm sè nµy b»ng f(2) = √ 3 . V× vËy ph¬ng tr×nh (1) cã nghiÖm khi vµ chØ khi √ m ³ √ 3 hay m ³3. b) m = 9 th× f(3) = 3 = √ 9 nªn víi m = 9 yh× hÖ cã nghiÖm duy nhÊt lµ (x ; y) = (3 ; 3) . Gi¶i hÖ ph¬ng tr×nh :. 26. VÝ dô 4 :. ¿. 3. Gi¶i: §Æt. 3. 2( x + y)=3 ( √ x2 y + √ xy 2 ) 3 3 √ x + √ y=6 ¿{ ¿ 3 3 u= √ x , v=√ y thì hệ đã cho trở thành.

<span class='text_page_counter'>(75)</span> u+ v=6 2(u 3+ v 3)=3 uv (u+ v) ⇔ ¿ u+v =6 3 u +v 3=9 uv ¿ ⇔ ¿ u+v =6 2 2 (u+ v)(u − uv+ v )=9 uv ⇔ ¿ ¿ u+v =6 2 u+ v ¿ −3 uv ¿=9 uv ¿ ¿ ¿ ⇔ ¿ ¿ u+v =6 ¿ 6 (36 −3 uv )=9 uv ¿ (u+ v )¿. VËy u , v lµ hai nghiÖm cña ph¬ng tr×nh : X2 - 6X + 8 = 0 .Suy ra (u ; v) = (2 ; 4) hoÆc (u ; v) = (4 ; 2) . V× vËy (x ; y) = (8 ; 64) hoÆc (x ; y) = (64 ; 8) . Hệ phơng trình đã cho có hai nghiệm : (x ; y) = (8 ;64) vµ (x ; y) = (64 ; 8) .. VI. Bµi TËp : 27.. Gi¶i hÖ ph¬ng tr×nh :. a). 28.. ¿ x+ √ √ y +1=1 √ x+1+ √ y=1 ¿{ ¿. b). ¿ √ x + √ y+ √ x − √ y=2 √ y+ √ x − √ y − √ x =1 ¿{ ¿. Gi¶i hÖ ph¬ng tr×nh :. ¿ 1 1 x1− y √ x 2 −1y 2 y − x √ x 2x− y 2 y ¿ √ 4y-1 + 2¿ − + 2− =2 ¿ ¿¿¿ e ¿ ¿ 2 +2 =b +¿ a ¿ x+ y=√ 4z-1¿ y + z= √ 4x-1 ¿ z + dx= ¿¿ b=2¿ ¿ ¿ =a y √x √√y1− x 2 + yx 2 √ 1− x +y y z. √. √. √ √ √.

<span class='text_page_counter'>(76)</span> 29.. Tìm m để hệ phơng trình. ¿ 2 x+ √ y −1=m 2 y − √ x −1=m ¿{ ¿. cã nghiÖm .. Chuyên đề BÊt Ph¬ng tr×nh - HÖ bÊt ph¬ng tr×nh. I.VÝ dô. VÝ dô 1. Gi¶i bÊt ph¬ng tr×nh: 1< 1+ x ≤ 2 . 1−x. Gi¶i :. §iÒu kiÖn x ¹ 1. Bất phơng trình đã cho tơng đơng với hệ bất phơng tr×nh sau: ¿ 1+ x 1< 1−x 1+ x ≤2 1− x ¿{ ¿. Ta cã : (1)⇔. (1) ( 2). 1+ x 2x −1>0 ⇔ >0 ⇔ 1−x 1−x ¿ 2 x <0 1− x<0 ¿ ¿ ¿ 2 x >0 ¿ ¿ 1− x>0 ¿ ¿ ¿.

<span class='text_page_counter'>(77)</span> (2)⇔. VÝ dô 2:. 1+ x 3 x −1 3 x −1 −2 ≤ 0 ⇔ ≤0⇔ ≥0⇔ 1−x 1−x x −1 x>1 ¿ 1 x≤ 3 ¿ ¿ ¿ ¿ ¿. Vậy mghiệm của bất phơng trình đã cho là : 0< x ≤ 1 3 .. Gi¶i bÊt ph¬ng tr×nh : (x2 + 4x + 10)2 - 7(x2 + 4x + 11) + 7 < 0 .. Gi¶i :. §Æt y = x2 + 4x + 10 , ta cã bÊt ph¬ng tr×nh : y2 - 7(y+1) < 0 Û y(y - 7) < 0 Û 0 < y < 7 Þ 0 < x2 + 4x +10 < 7 (1) 2 V× x + 4x +10 = (x + 2) 2 + 6 ³ 6 víi mäi gi¸ trÞ cña x nªn : (1) Û x2 + 4x +10 < 7 Û x2 + 4x +3 < 0 Û -3 < x < -1 . VËy nghiÖm cña bÊt ph¬ng tr×nh lµ : -3 < x < -1 .. VÝ dô 3 :. Víi nh÷ng gi¸ trÞ thùc nµo cña x th× bÊt ph¬ng tr×nh nào sau đây đợc nghiệm đúng 1− √1+2 x ¿ 2. Gi¶i :. ¿ ¿ 4 x2 ¿. .. Điều kiện để bất phơng trình có nghĩa : ¿ 1+2 x ≥ 0 1 ≠ 1+ 2 x ⇔0≠ x≥ − ¿{ ¿. Ta cã :. 1 2. (1).. 2 x(1+ √1+2 x) 2x = =−(1+ √ 1+ 2 x ) . 1 −(1+2 x ) 1 − √ 1+2 x.

<span class='text_page_counter'>(78)</span> 2. Nªn :. 1− √ 1+2 x ¿ ¿ 1+ √1+2 x ¿2=2+2 x+2 √ 1+2 x . ¿ 2 4x ¿. Do đó bất phơng trình đã cho trở thành : ¿ 2+2 x +2 √ 1+ 2 x <2 x+ 9⇔ 2 √1+2 x<7 ⇔ 4 (1+2 x )<49 45 ⇔x< ⋅ 8 ¿. Kết hợp với (1) ta có nghiệm của bất phơng trình đã cho lµ : ¿ 1 45 − ≤ x< 2 8 x≠0 ¿{ ¿. VÝ dô 4 :. Gi¶i hÖ bÊt ph¬ng tr×nh :. Gi¶i : ( I )⇔ x ( x − 2)≤0 (x − 1)(x −2)≤ 0 ⇔ ¿0≤ x≤2 1≤ x ≤ 2 ⇔1≤x ≤2 ¿{. VÝ dô 5 :. ¿ x 2 −2 x ≤ 0 x 2 −3 x +2 ≤0 ¿{ ¿. (I). .. VËy nghiÖm cña hÖ bÊt ph¬ng tr×nh lµ : 1≤ x ≤ 2. Gi¶i :. ¿ 2 x +5 x +4 <0 (1) 3 2 Gi¶i hÖ bÊt ph¬ng tr×nh : x +3 x − 9 x − 10>0 (2) ¿{ ¿. Ta cã : (1) Û -4 < x < -1 . MÆt kh¸c : x3 + 3x2 - 9x -10 = (x2 +5x + 4)(x - 2) - 3(x + 1) + 1 ..

<span class='text_page_counter'>(79)</span> Víi mäi x Î (-4 ; -1) th× (x2 +5x + 4)(x - 2) - 3(x + 1) + 1 > 0 , tức là với mọi x ẻ (-4 ; -1) đều thoả mãn bất phơng tr×nh (2) . Vậy nghiệm của hệ bất phơng trình đã cho : -4 < x < -1 .. II. Bµi tËp :. 1. Gi¶i c¸c bÊt ph¬ng tr×nh sau :. ¿ 11x − 5 x +6 1 2 1 −2 x2 2-x 1 −2 x 1- √ 1-4xx2 − 5 x+ 4 2 a 2 <x¿b¿ − 2 h ≤x ¿ 3+ √ x¿ −3 d ¿ x +5>3 x > +7 ¿ i¿ ¿ e ¿ <3 ¿ k ¿ ≤√ x+ 1 x − x +1 x +1 x x +5 x +6 x 3 + x 2 x3 −3 x 2 x2 − 4 2. |. |. 2. Gi¶i hÖ bÊt ph¬ng tr×nh : ¿ |x − 4 x|<5 | x+1|< 3 ¿{ ¿ 2. 3. Xác định tất cả các số thực x thoả mãn bất phơng trình 1 √ 3− x − √ x +1> . 2 4. Gäi u lµ nghiÖm d¬ng cña bÊt ph¬ng tr×nh x2 - 2px - q2 > 0 vµ gäi v lµ nghiÖm cña bÊt ph¬ng tr×nh x2 - 2px - q2 < 0 Chøng minh r»ng : u+v>p+q.. Bất đẳng thức Chuyên đề 1. Mét sè ph¬ng ph¸p chứng minh bất đẳng thức I. Phơng pháp dựa vào định nghĩa 1.kiÕn thøc c¬ b¶n. A³ BÛA-B³0 Chú ý các hằng đẳng thức :  a2 + 2ab + b2 = (a + b)2 ³ 0 .  a2 + b2 + c2 + 2ab + 2ac + 2bc = (a + b + c)2 ³ 0 .. 2.VÝ dô : VÝ dô 1 : Chøng minh r»ng víi mäi x , y ta lu«n cã :. ¿ 2 y a x ¿2 + ≥ xy ¿ b ¿ x2 + y 2 +1≥ xy+ x + y ¿ c ¿ x 4 + y 4 ≥ xy3 + x 3 y ¿ 4.

<span class='text_page_counter'>(80)</span> Gi¶i : a) XÐt hiÖu x2 +. VËy. 2. y ≥ xy 4. x2 +. y2 4 x 2 + y 2 − 4 xy 1 ( 2 − xy= = 2 x− y) ≥0 4 4 4. , dÊu “ = “ x¶y ra khi vµ chØ khi 2x = y .. ¿ 1( 2 2 b x ¿ + y +1 −(xy + x+ y)= 2x +2 y + 2− 2 xy − 2 x − 2 y ) ¿ 2 VËy x 2+ y 2 +1 ≥ xy + x+ y . ¿ x − y ¿2 (x 2+ xy + y 2) ¿ 2 2 y 3 y x-y ¿2 x+ + ≥0 2 4 2. 2. [( ) ]. c x ¿4 + y 4 −( xy 3+ x 3 y)=x ( x3 − y 3 )− y (x 3 − y 3)¿ VËy x 4 + y 4 ≥ xy 3+ x3 y .. =¿. VÝ dô 2 : Cho a < b < c < d , h·y s¾p xÕp thø tù t¨ng dÇn c¸c sè sau. x =(a + b)(c + d) ; y = (a + c)(b + d) ; z = (a + d)(b + c) .. Gi¶i :. XÐt hiÖu : y - x = (a + c)(b + d) - (a + b)(c + d) = ab + bc + ad + cd - ac - bc - ad - bd = b(a - d) - c(a - d) = (a - d)(b - c) > 0 (v× a < b < c < d) . Suy ra : y > x . T¬ng tù xÐt hiÖu : z - y = (a + d)(b + c) - (a + c)(b + d) = (a - b)(c - d) > 0 Suy ra : z > y . VËy x < y < z . : Cho abc = 1 vµ a3 > 36 Chøng minh r»ng :. VÝ dô 3. 2. a +b 2+ c 2> ab+ bc+ca 3. Gi¶i : 2. 2. .. 2. a a a +b 2+ c2 −ab − bc − ca= + +b2 +c 2 −ab − bc − ca 3 4 12 2 2 a a 2 2 = + b +c − ab −ac +2 bc + −3 bc 4 12 2 a 1 = − b −c + ( a3 −36 ) > 0 2 12a. ( (. (V× abc = 1 vµ a > 36 nªn a > 0) . 3. 2 VËy : a +b 2+ c 2> ab+ bc+ca .. 3. ). ).

<span class='text_page_counter'>(81)</span> II.Ph¬ng ph¸p sö dông tÝnh b¾c CÇu : 1.kiÕn thøc c¬ b¶n . ¿ A ≥B B ≥C ⇒ A≥C ¿{ ¿.  0 £ x £ 1 Þ x2 £ x ( v× x -x2 = x(1 - x) ³ 0). 2.VÝ dô : Ví dụ 4 : Cho a , b , c là độ dài ba cạnh của một tam giác có. Gi¶i :. chu vi b»ng 2 . Chøng minh r»ng a2 + b2 + c2 + 2abc < 2. NÕu a ³ 1 th× b + c > a ³ 1 Þ a + b + c > 2 , v« lý ! VËy 0 < a < 1 . T¬ng tù 0 < b < 1 ; 0 < c < 1 . Ta cã :(1 - a)(1 - b)(1 - c) = 1- a - b - c + ab + ac + bc -abc > 0 Þ abc < ab + ac + bc - 1 (v× a + b + c = 2 ) (1) Mµ 4 = (a + b + c)2 = a2 + b2 + c2 + ab + ac + bc Þ ab + ac + bc = 2 - 1 ( a2 + b2 + c2) (2) 2 Tõ (1) vµ (2) suy ra : abc < 2 - 1 ( a2 + b2 + c2) Þ a2 + b2 + c2 + 2abc 2 <2 Cho 0 < a , b , c , d < 1 chøng minh r»ng : (1 - a)(1 - b)(1 - c)(1 - d) > 1 - a - b - c - d .. VÝ dô 5 :. Gi¶i :. Ta cã (1 - a)(1 - b) = 1 - a - b + ab > 1 - a - b (1) . V× 1 - c > 0 nªn : (1 - a)(1 - b)(1 - c) > (1 - a - b)(1 - c) (2) . (1 - a - b)(1 - c) = 1 - a - b - c + c(a + b) > 1 - a - b - c (3) . Tõ (2) vµ (3) suy ra : (1 - a)(1 - b)(1 - c) > 1 - a - b - c . Suy ra : (1 - a)(1 - b)(1 - c)(1 - d) > (1 - a - b - c)(1 - d) >1 - a - b - c - d + d(a + b + c) > 1 - a - b - c - d. ( v× d(a + b + c) > 0) VËy : (1 - a)(1 - b)(1 - c)(1 - d) > 1 - a - b - c - d . Cho 0 £ a , b , c £ 2 tho¶ m·n a + b + c = 3.Chøng minh : a2 + b2 + c2 £ 5 .. VÝ dô 6 :. Gi¶i :.

<span class='text_page_counter'>(82)</span> V× a + b + c = 3 nªn Ýt nhÊt mét trong ba sè a , b , c kh«ng nhá h¬n 1 , gi¶ sö a ³ 1. V× 1£ a £ 2 nªn (a - 1)(a - 2) = a2 - 3a + 2 £ 0 Þ a(3 -a) ³2 Suy ra2 : ab2+ bc2 + ca = a(b + 2c) + bc = a(3 - a) + bc ³ 2 . VËy a + b + c = (a + b + c) - 2(ab + bc + ac) (1) = 9 - 2(ab + bc + ac) £ 5 ( theo (1)) VËy : a2 + b2 + c2 £ 5 .. III.Phơng pháp biến đổi tơng đơng 1.kiÕn thøc c¬ b¶n. Ta biến đổi bất đẳng thức cần chứng minh tơng đơng với bất đẳng đúng hoặc bất đẳng thức đã đợc chứng minh .. 2.VÝ dô : VÝ dô 7 :. a) Víi a , b , c > 0 , chøng minh :. a b c 1 1 1 + + ≥2 + − bc ca ab a b c. (. ). .. b) Cho a ³ c > 0 , b ³ c . Chøng minh : √ c (a − c)+ √ c (b −c )≤ √ ab. Gi¶i :. a b c 1 1 1 + + ≥2 + − ⇔ a2 +b 2+ c2 ≥2 (bc +ac − ba ) ( abc>0) ¿ bc ca ab a b c VËy a + b + c ≥ 2 1 + 1 − 1 bc ca ab a b c. (. a. ). (. ). 2. b √ c (a − c)+ √ c (b − c)≤ √ ab ⇔ ( √ c( a −c )+ √ c( b −c ) ) ≤ ab ¿ VËy √ c (a − c)+ √ c (b −c )≤ √ ab. Chú ý : Nếu ta biến đổi từ một bất đẳng thức đúng về bất. đẳng thức cần chứng minh thì chỉ cần sử dụng các phép biến đổi hệ quả . Cho a , b , c lµ ba sè tuú ý thuéc ®o¹n [0 ; 1] . Chøng minh r»ng : a(1 - b) + b(1 - c) + c(1 - a) £ 1.. VÝ dô 8 :. Gi¶i :. Ta cã 0 £ (1 - a)(1- b)(1 - c) = 1 - a - b - c + ab + bc + ca abc Þ 1 ³ a + b + c - ab - bc - ca + abc ³ a + b + c - ab - bc ca Mµ a + b + c - ab - bc - ca = a(1 - b) + b(1 - c) + c(1 - b) . VËy a(1 - b) + b(1 - c) + c(1 - a) £ 1 .. IV.Phơng pháp đánh giá đại diện 1.kiÕn thøc c¬ b¶n. ⇔ c (a −c )+ c (.

<span class='text_page_counter'>(83)</span> Nếu một bất đẳng thức có chứa những biểu thức có dạng tơng tự nhau thì ta có thể chứng minh bất đẳng thức đó bằng cách đánh giá một biểu thức đại diện .. 2.VÝ dô VÝ dô 8 : Chøng minh r»ng víi ba sè d¬ng a , b , c bÊt kú ta lu«n cã :. 3. 3. 3. a b c a+b+ c + 2 + 2 ≥ 2 2 2 2 3 a +ab+b b + bc+c c +ca +a. Gi¶i :. Các số hạng vế trái tơng tự nhau nên ta có thể nghĩ đến phơng pháp đánh giá đại diện một số hạng . Ta cần chứng minh : 3. a 2 a −b ≥ 2 2 3 a +ab+b. (1). Ta cã (1) Û 3a3 ³ (2a - b)(a2 +ab + b2) Û a3 + b3 -a2b -ab2 ³ 0 Û (a + b)(a2 - ab + b2) - ab(a + b) ³ 0 Û (a + b)(a2 - 2ab + b2) ³ 0 Û (a + b)(a - b)2 ³ 0 Bất đẳng thức cuối luôn đúng với a , b dơng nên bất đẳng thức (1) đúng , dấu “ =” xảy ra khi và chỉ khi a = b. T¬ng tù ta còng cã : b3 2b−c ≥ 2 2 3 b +bc +c 3 c 2c−a ≥ 2 2 3 c +ca +a. (2) (3). Céng (1) , (2) , (3) theo tõng vÕ ta cã : 3. 3. 3. a b c a+b+ c + 2 + 2 ≥ 2 2 2 2 3 a +ab+b b + bc+c c +ca +a. DÊu b»ng x¶y ra khi vµ chØ khi a = b = c .. V.Phơng pháp sử dụng các bất đẳng thức phụ 1.kiÕn thøc c¬ b¶n x2 + y 2 ≥2|xy| 2. ( x+ y ) ≥ 4 xy 1 1 4 + ≥ ( x , y >0) x y x+ y. 2.VÝ dô. 2. 2. x + y ≥2 xy 1 x+ ≥ 2, víi x >0 x 2 x+ y ¿ ¿ ¿ ¿ 1 4 ≥ xy ¿. ..

<span class='text_page_counter'>(84)</span> VÝ dô 9 : a) Cho a , b , c ³ 0 vµ a + b + c = 1 . Chøng minh r»ng ;. a + 2b + c ³ 4(1 - a)(1 - b)(1 - c) . b) Cho x , y > 0 vµ x + y - z = 1 . Chøng minh r»ng : x + y ³ 16xyz c) Cho a , b , c > 0 . Chøng minh r»ng : 1. 1 1 + a b. +. 1. 1 1 + b c. +. 1. 1 1 + c a. ≤. a+b+c 2. d) Hai sè d¬ng a , b tho¶ m·n ab > a + b . Chøng minh r»ng : a+b>4 e) Cho a , b , c là độ dài ba cạnh của tam giác có chu vi 2p . Chøng minh r»ng : 1 1 1 1 1 1 + + ≥2 + + p − a p −b p − c a b c. (. ). f) Cho 4 sè d¬ng a , b , c , d . Chøng minh r»ng : a b c d + + + ≥2 b+c c +d a+d a+ b. g) Cho hai sè d¬ng a , b vµ a + b = 1. Chøng minh r»ng : 1 1 + 2 2 ≥6 ab a + b. Gi¶i : a). áp dụng bất đẳng thức 4xy Ê (x + y)2 ta có : 4(1 - a)(1 - b)(1 - c) = [4(b + c)(1 - c)](1 - b) £ (1 + b)2 (1 - b) £ (1 + b)(1 - b)2 £ 1 + b = a + 2 b +c VËy a + 2b + c ³ 4(1 - a)(1 - b)(1 - c) . DÊu “ = “ x¶y ra khi vµ chØ khi a= 1 , b=0 , c= 1 . 2. 2. áp dụng bất đẳng thức 4xy Ê (x + y)2 ta có : 16xyz £ 4z(y + x)2 2 MÆt kh¸c (2z + 1) ³ 0 Û 4z2 + 4z + 1³ 0 Û 4z(z + 1) £ 1 Û 4z(x + y) £ 1Û 4z(x + y)2 £ x +y VËy x + y ³ 16xyz . c) Ta cã : b). ( a+b )2 ≥ 4 ab ⇒. T¬ng tù :. 1. 1 ≤ (b+c ) 1 1 4 + b c. ab 1 1 1 ≤ (a+b)⇒ ≤ (a+ b) a+b 4 1 1 4 + a b (2). (1).

<span class='text_page_counter'>(85)</span> 1. 1 ≤ (c+ a) 1 1 4 + c a. (3). Céng vÕ theo vÕ (1) , (2) ,(3) ta cã : 1. 1 1 + a b. +. 1. 1 1 + b c. +. 1. ≤. a+b+c 2. 1 1 + c a d) Tõ ab> a+b ⇒a> 1+ a vµ b>1+ b ⇒ a+b>2+ a + b b a b a Mµ a + b ≥2 ⇒ a+b> 4 . b a e) áp dụng bất đẳng thức 1 + 1 ≥ 4 ( x , y> 0) x y x+ y. ( ). cã :. 1 1 4 4 + ≥ = ; p − a p −b 2 p − a −b c 1 1 4 4 + ≥ = ; p − b p −c 2 p −b − c a 1 1 4 4 + ≥ = ; p − c p − a 2 p −c −a b ¿ 1 1 1 1 1 1 VËy : 2 + + ≥4 + + p− a p − b p −c a b c 1 1 1 1 1 1 ⇒ + + ≥2 + + ⋅ p − a p − b p −c a b c ¿ 2 x+ y ¿ ¿ ¿ áp dụng bất đẳng thức ta cã : 1 4 ≥ xy ¿ a+ b+c +d ¿2 ¿ (1) ¿ a+ b+c +d ¿2 ¿ ¿ ¿ a(d +a)+c (b +c) a c a2 +c 2+ ad+ bc + = ≥4⋅ ¿ b+c d +a ( b+c )( d +a). (. f). ) ( (. ). ). LÊy (1) céng (2) vÕ theo vÕ ta cã : a+b +c +d ¿2 ¿ a b c d a2+ b2 +c 2 +d 2+ ad+ bc+ ab+cd + + + ≥4 ⋅ ¿ b+c c +d a+d a+ b. ta.

<span class='text_page_counter'>(86)</span> MÆt kh¸c ta cã : (a - c)2 + (b - d)2 ³ 0 Û a2 + b2 + c2 + d2 - 2ac - 2bd ³ 0 Û 4(a2 + b2 + c2 + d2 + ad +bc+ab+cd)³(a + b + c + d)2 a+b+c+ d ¿2 ¿ ¿ Û 4 ( a2+ b2 +c 2 +d 2+ ad+bc+ ab+cd ) ¿ a b c d + + + ≥2 . VËy : b+c c +d a+d a+ b. áp dụng bất đẳng thức 4ab Ê (a + b)2 ta có : 1 1 ab ≤ ⇒ ≥ 4 .. g). 4 ab 1 1 4 áp dụng bất đẳng thức + ≥ (x , y> 0) ta cã : x y x+ y 2 a+ b ¿ ¿ ¿ 1 1 1 1 1 4 + 2 2= + + 2 2 ≥ 2+ ¿ ab a + b 2 ab 2 ab a +b 1 1 + 2 2 ≥6 . VËy : ab a + b. (. ). VI.Ph¬ng ph¸p ph¶n chøng. 1.kiÕn thøc c¬ b¶n Giả sử cần chứng minh một khẳng định “ A “ là đúng . Ta giả sử ngợc lại là khẳng định “A” không đúng rồi dùng phép suy luận lôgic để suy ra điều vô lý . Thế thì khẳng định “A” đúng .. 2.VÝ dô VÝ dô 10 : Cho 0 < a , b , c < 1 . Chøng minh r»ng cã Ýt nhÊt một trong các bất đẳng thức sau là sai :. Gi¶i :. ¿ 1 1 1 a(1 − b)> ; b (1− c )> ; c(1 − a)> ⋅ 4 4 4 ¿. Giả sử cả ba đẳng thức đều đúng , nhân các vế tơng ứng của ba bất đẳng thức ta có : abc(1 - a )(1 - b)(1 - c) > 1 64 (1) . Nhng : 1 1 2 1 2 a(a −1)=a − a = − a − ≤ 4 2 4 2 1 1 1 b(b −1)=b −b 2= − b − ≤ 4 2 4. ( ) ( ). . ..

<span class='text_page_counter'>(87)</span> 1 1 2 1 2 c (c −1)=c −c = − c − ≤ 4 2 4 Nªn : [ a(1 − a) ][ b(1− b) ][ c (1 −c ) ] ≤ 1 64. ( ). .. VËy : abc(1 - a )(1 - b)(1 - c) = [ a(1 − a) ][ b( 1− b) ][ c (1 −c ) ] ≤ 1 . 64 §iÒu nµy m©u thuÈn víi (1) , m©u thuÈn nµy chøng tá cã Ýt nhÊt mét trong ba bất đẳng thức đã cho là sai . Cho tam thøc bËc hai f(x) = x2 + bx + c . Chøng minh r»ng víi mäi gi¸ trÞ cña a , b trong ba sè |f(0)| , |f(1)| , |f(-1)| cã Ýt nhÊt mét sè lín h¬n hay b»ng 1 .. VÝ dô 11 :. 2. Gi¶i :. Giả sử ba số |f(0)| , |f(1)| , |f(-1)| đều nhỏ hơn 1 , tức là 2. 1 1 − < f (0)=b< 2 2 1 1 − < f (1)=1+a+b < 2 2 1 1 − < f (− 1)=1 − a+b< 2 2. (1) (2) (3). Céng c¸c vÕ t¬ng øng cña (2) vµ (3) ta cã : 1 1 1+b< ⇒b <− 2 2. M©u thuÈn víi (1) , m©u thuÈn nµy chøng tá trong ba sè | f(0)| , |f(1)| , |f(-1)| cã Ýt nhÊt mét sè lín h¬n hay b»ng 1 .. VII.. 2. Ph¬ng ph¸p lµm tréi 30.. kiÕn thøc c¬ b¶n a a a+c a , b>0 vµ <1 thi < b b b+c. 31. VÝ dô VÝ du 12 : a) Cho 3 sè d¬ng a , b , c . Chøng minh r»ng : 1<. Gi¶i : a). a b c + + <2 a+b b+c c +a. . b) Cho 4 sè d¬ng a , b , c , d . Chøng minh r»ng : a+ b b+c c +d d +a + + + , kh«ng lµ sè a+b+ c b+c +d c+ d+ a d + a+b nguyªn . Ta cã. a a a a+c <1 ⇒ < < a+b a+b+ c a+b a+b+ c. ..

<span class='text_page_counter'>(88)</span> T¬ng tù :. b b b+ a < < a+b+ c b+c a+b+ c c c c +b < < a+b+ c c+ a a+b+ c. Cộng các vế của bất đẳng thức trên ta có : a b c 1< + + <2 (®pcm) .. a+b b+c c +a Ta cã a+ b <1 ⇒ a+ b < a+ b < a+b+ d a+b+ c a+b+ c+ d a+b+ c a+b +c +d. b) T¬ng tù :. b+ c b+c b +c +d < < a+b+ c+ d b+c +d a+b+ c+ d c +d c+d c+ d+ b < < a+b+ c+d c +d +a a+b+ c+d d +a d+ a d +a+ c < < a+b+ c+ d d +a+b a+ b+c +d. Cộng vế theo vế các bất đẳng thức trên ta có :. a+ b b+c c +d d +a + + + <3 a+b+ c b+c +d c+ d+ a d + a+b a+ b b+c c +d d +a + + + kh«ng thÓ lµ sè a+b+ c b+c +d c+ d+ a d + a+b 2<. VËy nguyªn .. VÝ dô 13 : a) Víi sè n nguyªn d¬ng lín h¬n 1 . Chøng minh r»ng :. 1 1 1 1 + 2 +⋅+ 2 < 2− 2 n 1 2 n 1 1 1 5 + +⋅+ 2 < a.2 12 22 n 3 b) Cho d·y sè a1=1, a 2= 1 ,⋅⋅ , an=1+ 1 +⋅+ 1 .Chøng minh 2 2 n. a.1. r»ng. 1 1 1 + +⋅+ <2 a1 2 a2 na n 2. 2. víi mäi n > 1 .. 2. Cho d·y sè a1=1, a 2= 1 , ⋅⋅, an=1+ 1 + 1 +⋅+. c). . Chøng minh r»ng :. 3 3 5 1 1 1 + +⋅ + <2 . a1 3 a2 (2 n −1)a n 2. Gi¶i :. 2. 1 2 n −1. 2. 1 1 1 1 < = − do đó : 2 k −1 k k k (k −1) 1 1 1 1 1 1 1 1 1 + 2 +⋅+ 2 < 1+ 1 − + − + ⋅ − =2 − 2 2 2 3 n −1 n n 1 2 n. a) a.1 Víi k > 1 ta cã :. ( )(. ) (. ).

<span class='text_page_counter'>(89)</span> VËy : a.2. 1 1 1 1 + 2 +⋅+ 2 < 2− 2 n 1 2 n. Víi. k. .. >. 1. ta. 1 4 4 4 = 2< 2 = do đó 2 k 4 k 4 k −1 (2 k −1)(2 k +1) 1 1 1 <2 − 2 2 k −1 2 k +1 k. (. cã. :. ). Suy ra : 1 1 1 1 1 1 1 1 1 + 2 +⋅+ 2 < 1+ 2 − +2 − +⋅+2 − 2 3 5 5 7 2 n −1 2 n+1 1 2 n 1 1 2 5 =1 −2 − <1+ = 3 2n+ 1 3 3 1 1 1 5 VËy : 2 + 2 +⋅+ 2 < . 1 2 n 3. (. ) (. ) (. (. b). ). ). Víi k ³ 2 ta cã :. 1 1 < (v× ak > ak ka k kak − 1 a k 1 a k − ak −1= ) . k. ⇒. ). - 1. 2. 1 1 1 < − (V× ka k a k− 1 ak 2. Do đó : 1 1 1 1 1 1 1 1 1 1 + +⋅+ <1+ 1 − + − + − +⋅+ − a1 2 a2 na n a1 a1 a 2 a2 a3 an − 1 an 2. 2. (. 2. )(. =1+. )(. ) (. ). ( a1 − a1 )=2 − a1 <2 1. n. n. 1 1 1 VËy : a + 2 a +⋅+ na <2 víi mäi n > 1 . 1 2 n 2. c). 2. 2. Ta cã : a k − ak −1= 1. ⇒ ak >a k −1 ; 2 k −1 a k − ak − 1 1 1 1 1 < = = − (2 k −1)a k (2 k −1)ak −1 a k ak ak −1 a k− 1 ak. .. 2. Do đó :. 1 1 1 1 1 1 1 1 1 + +⋅+ <1+ − + − +⋅+ − a1 3 a2 a2 a3 a3 a 4 a n− 1 an (2 n −1)a n 1 1 1 =1+ − =2 − <2 a2 a n an 1 1 1 <2 . VËy : a + 3 a +⋅+ (2 n −1)a n 1 2 2. 2. (. 2. (. 2. VIII.. 2. )(. ) (. ). 2. Ph¬ng ph¸p qui n¹p 32.. kiÕn thøc c¬ b¶n. Nội dung phơng pháp qui nạp để chứng minh một bất đẳng thức phô thuéc vµo sè tù nhiªn n ³ n0 nh sau :. ).

<span class='text_page_counter'>(90)</span>  Với số tự nhiên n = n0 thì bất đẳng thức đúng .  Mỗi khi bất đẳng đó đúng với số tự nhiên n = k ³ n0 ta chứng minh đợc nó cũng đúng với n = k + 1 . Khi đó bất đẳng thức đã cho đúng với mọi số tự nhiên n ³ n0 .. 33. VÝ dô VÝ dô 14 : Cho a. ³ 0 . Chøng minh r»ng víi mäi sè nguyªn. d¬ng n th×. √ a+√ a+⋅ +√ a < 1+√ 42 a+ 1 ⏟. (1) .. n dÊu c¨n. Gi¶i :. a+ √a+⋅+ √ a §Æt x n=√⏟. , n nguyªn d¬ng th×. n dÊu c¨n. x n+1= √a+ x n.  Víi n = 1 th× x 1=√ a= √ 4 a < 1+ √ 4 a+1 , vËy 2 2 bất đẳng thức đúng với n = 1 .  Gi¶ sö x n< 1+ √ 4 a+1 , n lµ sè nguyªn d¬ng 2. nào đó ta cần chứng minh x n+1 < 1+ √ 4 a+1 . 2 ThËt vËy 1+ 4 a+1 2+2 √ 4 a+1 x = a+ x < a+ √ = a+ n +1. n. √. 2. √. 4. 2. ( 4 a+1)+2 √ 4 a+1+1 √ 4 a+ 1+1 = √ 4 a+1+1 = 4 2 2 1+ √ 4 a+ 1 a+ √ a+⋅ + √ a < √⏟ , víi mäi sè 2 n dÊu c¨n =. VËy. √. √(. √. ). nguyªn d¬ng n Cho a , b lµ hai sè tuú ý tho¶ m·n ®iÒu kiÖn a + b ³ 0 . Chøng minh r»ng víi mäi sè nguyªn d¬ng n th×. VÝ dô 15 :. Gi¶i :. a+b n an +b n ≤ 2 2. ( ).  Víi n = 1 th×. (1). a+b 1 a1 +b1 = 2 2. ( ). a+b n an +b n ≤ 2 2. ( ). ,. đúng  Giả sử n = k ³ 1 thì bất đẳng thức (1) đúng tức là : a+b k ak +b k . ≤. (2). 2.

<span class='text_page_counter'>(91)</span> Ta sẽ chứng minh bất đẳng thức (1) cũng đúng với n = k a+b 2. k+1. ( ). +1 , tøc lµ ph¶i chøng minh :. a k+1 +b k+1 ≤ 2. .. V× a + b ³ 0 nªn a+b 2. k+1. k. a+b = 2. ( ) ( ). a+b ak +b k a+b ≤ 2 2 2. Do đó cần chứng minh k. k. k +1. a +b a+b a ≤ 2 2. k +1. +b 2. ⇔( a −b)( ak − b k ). (2) .. V× a , b cã vai trß nh nhau nªn cã thÓ gi¶ thiÕt a ³ b , mÆt khác từ a + b ³ 0 ị a ³ -b ị a ³ | b | do đó ak - bk ³ 0. Vậy bất đẳng thức (2) đã đợc chứng minh . n. VËy :. IX.. n. a+b a +b ≤ 2 2. ( ). n. đúng với mọi số nguyên dơng n .. Ph¬ng ph¸p sö dông ®iÒu kiÖn vÒ nghiÖm cña tam thøc bËc hai . 34.. kiÕn thøc c¬ b¶n Cho tam thøc bËc hai :. 35.. f(x) = ax2 + bx + c , a ¹ 0 .  f(x) cã nghiÖm khi vµ chØ khi D = b2 - 4ac ³ 0 .  NÕu D = b2 - 4ac < 0 th× a.f(x) > 0 víi mäi gi¸ trÞ cña x .  NÕu D = b2 - 4ac £ 0 th× a.f(x) ³ 0 víi mäi gi¸ trÞ cña x .. VÝ dô : VÝ dô 16 Cho (x ; y ; z) lµ nghiÖm cña hÖ ph¬ng tr×nh ¿ x + y + z 2=8 xy + yz+zx =4 ⋅ { ¿ 2. 2. Chøng minh r»ng :. ¿ 8 8 − ≤ x; y ; z≤ ⋅ 3 3 ¿. Gi¶i : Ta cã :. ¿ x 2+ y 2=8 − z 2 (1) xy=4 − z(x + y ) (2) ⋅ { ¿. Tõ (1) vµ (2) suy ra : 8 - z2 = (x + y)2 - 2xy = (x + y)2 - 8 + 2z(x + y) Þ (x + y)2 + 2z(x + y) + z2 = 16 Þ [x + y + z]2 = 42 Þx+y+z=±4.  NÕu x + y + z = - 4 th× x + y = - z - 4 , tõ (2) suy ra.

<span class='text_page_counter'>(92)</span> xy = 4 + z(4 + z) = ( z + 2)2 . VËy x , y lµ nghiÖm cña ph¬ng tr×nh X2 + (4 + z) X + (z + 2)2 = 0 . Ta ph¶i cã D = (4 + z)2 - 4(z + 2) = -z(3z + 8) ³ 0 8 ⇔ − ≤ z ≤ 0(3) . 3  NÕu x + y + z = 4 th× x + y = 4 - z , tõ (2) suy ra xy = 4 - z(4 - z) = ( z - 2)2 . VËy x , y lµ nghiÖm cña ph¬ng tr×nh X2 - (4 - z) x + (z - 2)2 = 0 . Ta ph¶i cã D = (4 - z)2 - 4(z - 2) = z(8 - 3z) ³ 0 8 ⇔ 0 ≤ z ≤ (4) . 3. Tõ (3) , (4) ta cã : − 8 ≤ z ≤ 8 3. 3. V× vai trß x , y , z nh nhau nªn ta cã − 8 ≤ x ; y ; z ≤ 8 . 3. VÝ dô 17 : Chøng minh r»ng víi mäi gi¸ trÞ x , y th×. 3. (x + y)2 - xy + 1 (x+ y) √ 3 Bất đẳng thức cần chứng minh tơng đơng x 2+ xy+ y 2 ≥ x √ 3+ y √ 3 ⇔ x 2+( y − √ 3) x + y 2 − y √ 3+1 ≥ 0 Vế trái của bất đẳng thức trên là một tam thức bậc hai đối với x nã cã hÖ sè cña x2 b»ng 1 vµ 2 2 Δ=( y − √ 3 ) −4 ( y 2 − y √ 3+1 ) =− ( √ 3 y − 1 ) ≤ 0, ∀ y ∈ R VËy x 2+( y − √ 3)x + y 2 − y √ 3+1 ≥ 0, ∀ x ; y ∈ R Tøc lµ : (x + y)2 - xy + 1 (x+ y) √ 3 .. Gi¶i :. Bµi tËp 1.. Cho ba sè thùc x , y , z bÊt kú . H·y chøng. minh : 2. minh r»ng :. x2 + y2 + z2 ³ | xy + yz + zx| . Cho ba sè d¬ng a , b , c kh¸c 0 . Chøng a2 b2 c 2 a b c + + ≥ + + b2 c 2 a2 b c a. 3.. .. Cho ba số a , b , c trong đó ít nhất hai số khác nhau . Đẳng thức sau đúng hay sai : a2 + b2 + c2 = | ab + bc + ca | ?. 4. Cho ba sè a , b , c bÊt kú . Chøng minh rằng trong ba bất đẳng thức : a2 +b 2 ≥. ( b +c )2 2 2 ( c +a )2 2 2 ( a+b )2 ; b +c ≥ ;c +a ≥ 2 2 2. đẳng thức đúng .. . cã Ýt nhÊt mét bÊt.

<span class='text_page_counter'>(93)</span> 5.. Chøng minh r»ng víi n lµ sè nguyªn d¬ng. bÊt kú th×. 1 1 1 1 + 2 + 2 +⋅+ 2 <1 ,65 . 2 1 2 3 n. A= 6.. Chứng minh rằng nếu a , b , c là độ dài ba c¹nh cña mét tam gi¸c th× biÓu thøc P = a2b + b2c + c2a + a2c + c2b + b2a - a3 - b3 - c3 - 2abc lu«n lu«n lµ sè d¬ng . 7. Víi ba sè d¬ng a , b , c bÊt kú , h·y chøng minh các bất đẳng thức sau : ¿ ab bc ab bc ca a3 +b 3 b3 +c 3 c 3 +a 3 a + ≥2 b . ¿ b ¿ + + ≥ a+ b+c . ¿ c ¿ + + ≥ a+b+ c . ¿ c a c a b 2 ab 2 bc 2ca. Chøng minh r»ng nÕu x , y , z ³ 0 th× : x(x - y)(x - z) + y(y - x)(y - z) + z(z - x)(z - y) ³ 0 . 9. Cho a , b , c Î [0 ; 2] cã tæng a + b + c = 3 . Chøng minh r»ng : a2 + b2 + c2 £ 5 . 10. Cho a , b , c > 1 vµ abc = 1 . Chøng minh r»ng : 8.. 11.. ab bc ac + 5 5 + 5 5 ≤1 5 5 a +b +ab b +c + bc a +c +ac. Cho a , b , c > 0 . Chøng minh r»ng. 3. 5 b − a 5 c − b3 5 a 3 − c 3 + + ≤ a+b+ c ab+3 b 2 bc+ 3 c 2 ca +3 a2. 12.. 13.. .. 3. 3. .. Cho a , b , c tho¶ m·n :. ¿ a+b+ c> 0 ab+ bc+ ca> 0 abc> 0 ¿{{ ¿. .. Chøng minh r»ng ba sè a , b , c d¬ng . Chøng minh r»ng nÕu x , y nguyªn d¬ng thì một trong hai bất đẳng thức sau là sai :. 1 1 1 1 ≥ + 2 x ( x + y ) √ 5 x ( x+ y )2 n 14.Chøng minh r»ng víi mäi sè nguyªn d¬ng n th× : 1+ 1 <3 . n 1 1 1 1 ≥ + xy √ 5 x 2 y 2. (. ). vµ. [. ]. .. ( ). 15.Chøng minh r»ng nÕu ph¬ng tr×nh 2x + (x + a)2 + (x + b)2 = c cã nghiÖm th× : 4c2 ³ 3(a2 + b2) - ab . 16.Cho x , y lµ hai sè thùc víi x , y > √ 2 . Chøng minh r»ng : x4 - x3y + x2y2 - xy3 + y4 > x2 + y2 . 8 17. Chøng minh r»ng : 4a - 2a7 + a6 - 3a4 + a2 - a + 1 > 0 , víi mäi a Î R . 18. Cho ba sè thùc a , b , c , d tho¶ m·n 0 £ a , b , c , d £ 1 . Chøng minh r»ng : a b c d + + + ≤3 . 2. bcd+1 cda +1 dab+1 abc+ 1.

<span class='text_page_counter'>(94)</span> 19.Cho k > 0 vµ ba sè thùc x , y , z sao cho :. ¿ xyz=k 3 xy + yz+zx < k (x+ y+ z) ¿{ ¿. .. Chứng minh rằng ba số x , y , z có đúng một số lớn hơn k . 20.Cho ba số a , b , c Ê 1 .Chứng minh rằng ít nhất một trong các bất đẳng thức sau là đúng a) a(ab + c) £ 1 . b) (2 - a)[(2 - a)b + 1] £ 1 . 21.cho ba sè a , b , c lµ c¸c d¬ng 1 1 1. a) Nếu a + b + c ³ 3 thì có bất đẳng thức sau không : a + b + c ≤ 3 . 1 1 1. b) Nếu a + b + c Ê 3 thì có bất đẳng thức sau không : a + b + c ≥ 3 . 22. a) Cho a2001 + b2001 > a2000 + b2000 . Chøng minh r»ng : a2002 + b2002 ³ a2001 + b2001 . b) Cho a1 , a2 , ... , an ; b1 , b2 , ... , bn ; c1 , c2, ... , cn lµ c¸c sè d¬ng vµ ¿ a1 +a 2+⋅+an =A b1 +b2 +⋅+ bn=B c 1+ c 2+⋅+c n=C ¿{{ ¿. . ABC. Chøng minh r»ng : { a 1 b1 c 1 , a 2 b 2 c 1 ,⋅⋅ , an bn c n } ≤ 3 n 23.Chøng minh r»ng víi mäi sè nguyªn d¬ng n th× : 1 1 1 + +⋅+ <2 2 3 √2 ( n+ 1) √ n. .. 24.Chøng minh r»ng víi mäi a , b , c , d d¬ng ta cã : a) 1< a + b + c <2 . b+c c +a a+b b) 1< a + b + c + d <2 . a+b+ c b+c +d c+ d+ a d +a+ b c) 1< a+ b + b+c + c +d + d +a <3 . a+b+ c b+c +d c+ d+ a d +a+ b. 25.Chøng minh r»ng víi mäi sè tù nhiªn n , n ³ 1 th× : n+1 ¿ n ¿ . ¿ √ 2+ √3 2+⋅+ √ ¿. 26.Cho ba sè d¬ng a , b , c tho¶ m·n abc = 1 . Chøng minh r»ng : 1 1 1 + + ≤1 . 1+ a+b 1+ b+c 1+c +a. ..

<span class='text_page_counter'>(95)</span> ¿ a<b< c a+b+ c=6 . ab+ bc+ ca=9 ¿{{ ¿. 27.Cho a , b , c lµ ba sè d¬ng tho¶ m·n :. Chøng minh: a <1< b <3 < c <4 . 28.Cho a,b,c, d dơng .Chứng minh rằng không thể đồng thời xảy ra ba bất đẳng thức :  a+b<c+d.  (a + b)(c + d) < ab + cd .  (a + b)cd < (c + d)ab . 29. Cho ba sè thùc bÊt kú x , y , z . Chøng minh r»ng : a) (x - y)2 + (y - z)2 + (z - x)2 £ 3(x2 + y2 + z2) . b) NÕu m lµ sè nhá nhÊt trong ba sè (x - y)2 , (y - z)2 , (z - x)2 th× m≤. x 2 + y 2 +z 2 2. .. 30. Cho x , y , z là các số thực đều lớn hơn -1 và thoả mãn : x3+ y3+ z3 ³ x2+ y2+ z2 . Chøng minh r»ng : x5 + y5 + z5 ³ x2 + y2 + z2 . 31.Cho c¸c sè thùc x , y , z tho¶ m·n : x £ y £ z . Chøng minh r»ng xy4 + yz4 + zx4 ³ yx4 + zy4 + xz4 . 32. Cho a , b , c Î [0 ; 1] . Chøng minh r»ng : a + b2 + c3 - ab - bc - ca £ 1 . 33. Cho x , y , z Î [0 ; 2] . Chøng minh r»ng : 2(x + y + z) - (xy + yz + zx) £ 4 . 34.Cho a , b , c > 0 tho¶ m·n a + b + c = 1 . Chøng minh r»ng : 6(ab + bc + ca) + a(b - c)2 + b(c - a)2 + c(a - b)2 £ 2 . 35.Cho x + y + z = 0 vµ x , y , z Î [-1 ; 1] . Chøng minh r»ng : x 2 + y4 + z 6 £ 2 . 36.Chứng minh rằng nếu hai số nguyên dơng m , n thoả mãn bất đẳng thức m m 1 . √ 7− >0 . Th× : √ 7− > n. n mn. Chuyên đề 2. Bất đẳng thức cauchy , bất đẳng thức bunhiacôpxki. i. Bất đẳng thức Cauchy. 1.Bất đẳng thức Cauchy Cho n sè kh«ng ©m a1 , a2 , ... , an . Ta cã : a1 +a2 +⋅+ an n ≥ √ a1 a2 ⋅⋅ an n. . DÊu “ = “ x¶y ra khi vµ chØ khi a1. = a2 = . . . = an . Bất đẳng thức Cauchy còn đợc gọi là bất đẳng thức về trung bình cộng và trung bình nhân ..

<span class='text_page_counter'>(96)</span> 2.VÝ dô : VÝ dô 1 : Cho x , y z > 0 . Chøng minh r»ng : ¿ x y 1 1 4 a + ≥ 2. ¿ b ¿ + ≥ .¿c¿ y x x y x+ y. Gi¶i :. DÊu b»ng x¶y ra khi nµo .. a) áp dụng bất đẳng thức Cauchy cho hai số dơng cã x y x y + ≥2 =2 y x y x. √. x y. 1 1 1 9 + + ≥ .¿ x y z x+ y+z. y x. vµ x y = y x. , dÊu “ = “ x¶y ra khi vµ chØ khi. ta hay. x=y b) áp dụng bất đẳng thức Cauchy ta có : 1 1 1 1 1 + ≥2 ⋅ =2 x y x y xy x+ y ≥ 2 √ xy. √. √. (1) (2). Nhân các vế tơng ứng của (1) và (2) thì đợc :. ( 1x + 1y )( x + y ) ≥2 √ xy1 ⋅2 √ xy=4. (3) .. Vì x + y > 0 nên (3) tơng đơng : 1 1 4 + ≥ , dÊu “ = “ x¶y ra khi vµ chØ khi x = y . x y x+ y c) áp dụng bất đẳng thức Cauchy ta có : 1 1 1 3 1 1 1 3 1 + + ≥3 ⋅ ⋅ =3 x y z x y z xyz 3 x + y + z ≥ 3 √ xyz. √. √. (4 ) (5). Nhân các vế tơng ứng của (4) và (5) thì đợc :. ( 1x + 1y + 1z ) ( x + y + z ) ≥3 √ xyz1 ⋅3 √ xyz=9 3. 3. (6) .. Vì x + y + z > 0 nên (6) tơng đơng : 1 1 1 9 + + ≥ , dÊu “ = “ x¶y ra khi vµ chØ khi x y z x+ y+ z z=y=z : Cho x , y , z ³ 0 vµ x + y + z £ 3 . Chøng minh r»ng. :. VÝ dô 2. Gi¶i :. x y z 3 1 1 1 + + ≤ ≤ + + 2 2 2 1+ x 1+ y 1+ z 2 1+ x 1+ y 1+ z. 2 a) Ta cã 2x £ 1 + x2 ⇒ 2 x 2 ≤ 1+ x 2 =1 ⇒ x 2 ≤ 1 .. 1+ x. 1+ x. 1+ x. 2. ..

<span class='text_page_counter'>(97)</span> DÊu “ = “ x¶y ra khi vµ chØ khi x2 + 1 = 2x Û x =1 . y 1 z 1 ≤ ≤ ; 2 2 1+ y 2 1+ z 2 x y z 3 + + ≤ . 2 2 2 1+ x 1+ y 1+ z 2. T¬ng tù ta cã : Do đó :. .. DÊu “ = “ x¶y ra khi vµ chØ khi x = y = z = 1 . b) ¸p dông kÕt qu¶ vÝ dô 1c) ta cã :. ( 1+1 x + 1+1 y + 1+1 z ) ( 1+ x +1+ y +1+ z ) ≥ 9. .. DÊu “ = “ x¶y ra khi vµ chØ khi 1 +x = 1 +y = 1 +z Û x =y=z . 9 V× vËy : 1 + 1 + 1 ≥ . 1+ x 1+ y 1+z. 3+x+ y+z. Mµ 3 + x + y + z £ 6 nªn :. 9 9 3 1 1 1 3 ≥ = ⇒ + + ≥ 3+ x+ y + z 6 2 1+ x 1+ y 1+ z 2. . DÊu “ = “ x¶y ra khi vµ chØ khi x = y = z vµ x + y + z = 3 tøc lµ khi x = y = z = 1 . : Chøng minh r»ng nÕu c¸c sè d¬ng a , b , c tho¶. VÝ dô 3. m·n ®iÒu kiÖn. Gi¶i :. 1 1 1 + + ≥2 1+ a 1+b 1+ c. , th× abc £ 1 . 8. Tõ 1 + 1 + 1 ≥2 ⇒ 1 ≥ 1 − 1 + 1− 1 = b + c 1+ a 1+b 1+ c 1+a 1+ b 1+c 1+ b 1+c áp dụng bất đẳng thức Cauchy ta có :. .. b c bc , dÊu “ = “ x¶y ra khi vµ chØ khi : + ≥2 1+b 1+c (1+b)(1+ c) b c = ⇔ b=c 1+b 1+ c VËy : 1 ≥ 2 bc (1) 1+a (1+b)(1+c). √. √. Chøng minh t¬ng tù ta cã : 1 ≥2 1+b 1 ≥2 1+ c. √ √. ac (1+a)(1+ c) ab (1+ a)(1+b). (2) (3). Nh©n c¸c vÕ t¬ng øng cña (1) , (2) , (3) ta cã :. 1 abc ≥8⋅ . (1+a)(1+b)(1+c ) (1+a)(1+b)(1+c ) V× a , b , c d¬ng nªn suy ra abc 1 , dÊu “ = “ x¶y ra khi 8 1 vµ chØ khi a = b = c = . 2.

<span class='text_page_counter'>(98)</span> II.Bất đẳng thức Bunhiacôpxki 1.Bất đẳng thức Bunhiacôpxki Víi hai bé n sè (a1 , a2 , . . . , an) vµ (b1 , b2 , . . . , bn) bÊt kú , ta lu«n cã (a1b1 + a2b2 + ... + anbn)2 £ (a12 + a22 + ...+an2) (b12 + b22 + ...+bn2) . DÊu “ = “ x¶y ra khi vµ chØ khi tån t¹i k sao cho ai = kbi (*) víi mäi i = 1 , 2 , ... , n . a1 a2 an = =⋅= b1 b 2 bn. (nếu bi ạ 0 với mọi i thì (*) đợc viết thành. .). Bất đẳng thức trên còn đợc gọi là bất đẳng thức Schwarz , hay bất đẳng thức Cauchy - Schwrt .. 2.VÝ dô : VÝ dô 1 : Cho a , b , c lµ ba sè kh¸c 0 . Chøng minh r»ng : ¿ 2 2 2 a b c a b c + + ≥ + + ⋅ b2 c 2 a 2 b c a ¿. Gi¶i :. áp dụng bất đẳng thức Bunhiacôpki ta có : ¿. (. 2. 2. 2. 2. |b| |c| |a| |b| |c| a b c ( 2 2 2 ) |a| + 2 + 2 1 +1 +1 ≥ ⋅1+ ⋅1+ ⋅1 = + + 2 |b| |c| |a| |b| |c| |a| b c a. ). (. ) (. 2. ). (1)⋅. ¿. Mặt khác , áp dụng bất đẳng thức Cauchy ta nhận đợc : |a| |b| |c| 3 |a| |b| |c| + + ≥3 ⋅ ⋅ =3 (2) . |b| |c| |a| |b| |c| |a| Tõ (1) vµ (2) suy ra :. √. ¿ 2 |a| |b| |c| |a| |b| |c| a b c 3 2 + 2+ 2 ≥ + + ≥3 + + |b| |c| |a| |b| |c| |a| b c a ¿ |a| |b| |c| a b c a2 b2 c 2 + + + + . + + ≥ |b| |c| |a| b c a b2 c 2 a2. (. Hay :. 2. 2. 2. )(. ) (. ). (1)⋅. DÊu “ = “ x¶y ra khi vµ chØ khi a = b = c . : Chứng minh rằng nếu a , b , c là độ dài các cạnh cña mét tam gi¸c cã p lµ nöa chu vi th× : √ p< √ p − a+ √ p − b+ √ p− c ≤ √ 3 p .. VÝ dô 2 . Gi¶i :. áp dụng bất đẳng thức Bunhiacôpki cho hai bộ số (1 , 1 , 1) vµ ( √ p − a , √ p − b , √ p −c ) ta cã :.

<span class='text_page_counter'>(99)</span> 2. ( 1⋅ √ p − a+1⋅ √ p −b+ 1⋅ √ p −c ) ≤ ( 12 +12 +12 ) ( p − a+ p −b+ p − c )=3 p hay : √ p − a+√ p− b+ √ p −c ≤ √3 p (1)  (2) √ p< √ p − a+ √ p − b+ √ p− c (2)⇔ p < p − a+ p −b+ p − c+ 2 √( p − a)( p− b) +2 √ ( p −a)( p − c)+2 √( p− b)( p −c ) 0<2 √( p −a)( p − b)+2 √ ( p − a)( p − c)+2 √( p −b)( p − c). hay: Bất đẳng thức này luôn luôn đúng nên bất đẳng thức (2) đúng . KÕt hîp víi (1) ta cã : √ p< √ p − a+√ p − b+ √ p− c ≤ √ 3 p . : C¸c sè kh«ng ©m x vµ y tho¶ m·n ®iÒu kiÖn x 3 + y3 =. VÝ dô 3 2.. Gi¶i :. Chøng minh r»ng : x2 + y2 £ 2 .. áp dụng bất đẳng thức Bunhiacôpki cho hai bộ số ( √ x ; √ y) vµ ( √ x3 ; √ y 3) ta cã : 2 2 (1) . ( x 2+ y 2) =( √ x √ x 3+ √ y √ y 3 ) ≤ (x+ y )(x 3 + y 3)=2(x+ y) Lại áp dụng bất đẳng thức Bunhiacôpxki một lần nữa ta có : (x + y)2 £ (12 + 12)(x2 + y2) = 2(x2 + y2) (2) . Tõ (1) vµ (2) suy ra : (x2 + y2)4 £ 4(x + y)2 £ 8(x2 + y2) . Hay : (x2 + y2)3 £ 8 tøc lµ x2 + y2 £ 2 . DÊu “ = “ x¶y ra khi vµ chØ khi : ¿ x + y 3 =2 √ x = √ x3 √ y √ y3 x y = 1 1 ⇔ x= y =1 ¿{{ ¿ 3. Bµi tËp. 1. Chøng minh r»ng nÕu a , b , c lµ ba c¹nh cña tam gi¸c ABC víi p lµ nöa chu vi th× : 1 1 1 1 1 1 + + ≥2 + + p − a p −b p − c a b c. (. ). 2. Chøng minh r»ng nÕu x , y , z lµ c¸c sè d¬ng th× x2 y2 z2 x+ y+z + + ≥ y+z x+z x+ y 2. 3. Chøng minh r»ng víi mäi sè d¬ng a , b , c , d > 0 th× a− d d −b b − c c − a + + + ≥0 d +b b+ c c+ a a+d. ..

<span class='text_page_counter'>(100)</span> 1. ≤ x 3+ y 3 ≤ 1 . 4. Cho x , y ³ 0 vµ x2 + y2 = 1 . Chøng minh √2 5. Hai sè d¬ng a , b cã tæng b»ng 1 . Chøng minh 1 1 a + 2 2 ≥6⋅ ab a + b. ¿ b¿. 2 3 + 2 2 ≥14 ⋅¿ ¿ ab a +b. 6. Ba sè kh«ng ©m a, b, c cã tæng b»ng 1.Chøng minh r»ng : a +2b +c ³ 4(1 - a)(1 - b)(1 - c) 7. Chøng minh r»ng víi mäi sè d¬ng a , b , c th× ¿ 1 1 1 1 1 1 1 + + ≤ + + ⋅ 2 a+ b+c a+ 2b +c a+ b+2 c 4 a b c ¿ ¿ 1 a3 3 1 a 8. Cho a , b > 0 . Chøng minh + +b ≥ + +b ⋅ a b a3 b3 ¿ ¿ x 3 y3 z 3 9. Chøng minh r»ng nÕu x , y , z > 0 th× + + ≥ x+ y + z ⋅ y 2 z2 x 2 ¿. (. ). 10.Cho a , b , c lµ sè ®o ba c¹nh cña mét tam gi¸c . Chøng minh r»ng : ( a + b - c)(a + c - b)(b + c - a)£ abc . 11.Cho c¸c sè d¬ng a , b , c , d . Chøng minh r»ng : 2. 2. 2. 2. ¿. a b c d 1 1 1 1 + + + ≥ + + + ⋅ b5 c 5 d 5 a5 a 3 b3 c 3 d3 ¿. 12.Tổng của 5 số không âm bằng đơn vị . Chứng minh rằng có thể xếp chúng trên một đờng tròn sao cho tổng của tất cả 5 tích của các cặp số đứng cạnh nhau không lớn hơn 1 5 13.Cho ba sè d¬ng a , b , c tho¶ m·n a + b + c = 1 . Chøng minh r»ng : ¿. √ 1− a+ √ 1− b+ √ 1− c ≤ 6 ⋅ ¿ 14.Cho n sè thùc a1 , . . . , an tho¶ m·n a12 + a22 + . . . + an2 = 3 . Chøng minh r»ng :. |. ¿ a1 a 2 a + +⋅+ n < √ 2⋅ 2 3 n+1 ¿. |. 15.Cho a > b ³ 0 . Chøng minh r»ng. 2. b +1¿ ¿ ≥ 3⋅ (a − b)¿ ¿ 4 a+ ¿.

<span class='text_page_counter'>(101)</span> 16.Chøng minh r»ng nÕu a , b d¬ng vµ a + b = 1 th×. 1 2 1 2 25 + b+ ≥ b a 2. ( )( ) a+. .. 17.Chøng minh r»ng víi mäi a , b , c d¬ng th× ¿ 1 1 1 ( 1 1 1 + + ≥ 1+ √ 2+ √3 ) + + ⋅ a b c a+ √ 2b+ √ 3 c b+ √ 2 c+ √3 a c+ √ 2 a+ √3 b ¿. (. ). 18.Chøng minh r»ng nÕu c¸c sè d¬ng a , b , c tho¶ m·n a + b + c £ 1 th× ¿ 1 1 1 + 2 + 2 ≥ 9⋅ 2 a +2 bc b +2 ac c +2 ab ¿. 19.Cho ba sè d¬ng tho¶ m·n a + b + c £ 3 . Chøng minh r»ng 1 1 1 3 + + ≥ . 1+ ab 1+ bc 1+ca 2 20.Cho ba sè thùc a , b , c n»m gi÷a 0 vµ 1 . Chøng minh r»ng : ¿ a b c + + +(1− a)(1− b)(1− c )≤ 1⋅ 1+b +c 1+a+ c 1+ a+b ¿. 21.Cho nsè d¬ng bÊt kú a1 , a2 , . . . , an > 0 . Chøng minh r»ng : ¿ n n (1+a1 )(1+a2 )⋅⋅ (1+a n) ≥ ( 1+ √ a1 a2 ⋅ ⋅ an ) ⋅ ¿. 22.Cho a , b , c > 0 vµ abc = 1 . Chøng minh r»ng : ¿ b+c c +a a+b + + ≥ √ a+ √ b+ √ c +3 ⋅ √a √b √c ¿. 23.Cho a , b , c , d > 0 vµ c 2 + d2 = (a2 + b2)3 . Chøng minh r»ng ¿ a3 b3 + ≥1 ⋅ c d ¿. 24.Chứng minh bất đẳng thức sau với a , b , c , d dơng tuỳ ý ¿ 1 1 1 p+q p +q p+q + + ≥ + + ⋅ a b c pa +qb pb+qc pc+ qa ¿. ứng dụng bất đẳng thức Chuyên đề 1.

<span class='text_page_counter'>(102)</span> áp dụng bất đẳng thức để tìm gi¸ trÞ línKiÕnnhÊt , nhá nhÊt thøc c¬ b¶n  Gi¶ sö f(x) £ k ( k lµ h»ng sè ) vµ dÊu " = " x¶y ra khi x = a th× gi¸ trÞ lín nhÊt cña f(x)lµ k khi x = a . Ký hiÖu Maxf(x) = k khi x = a .  Gi¶ sö f(x) ³ k ( k lµ h»ng sè ) vµ dÊu " = " x¶y ra khi x = a th× gi¸ trÞ nhá nhÊt cña f(x) lµ k khi x = a . Ký hiÖu Minf(x) = k khi x = a .. C¸c bµi to¸n. Bµi 1 : a) T×m gi¸ trÞ nhá nhÊt cña A = 2x2 - 6x + 1 . Tæng qu¸t t×m gi¸ trÞ nhá nhÊt cña P = ax2 + bx + c (a > 0 ) . b) T×m gi¸ trÞ nhá nhÊt cña B = x - x . c) T×m gi¸ trÞ nhá nhÊt cña C = 9x2 - 6x - 4 Gi¶i. 3x  1. +6. 2. 9 9 3 7 7  2   x  3 x     1 2  x    ³ 4 2 2 2 2  a) Ta cã A = 2( x2 - 3x ) + 1 = 2  7 3   VËy minA = 2 khi x 2 .  b  b b2  P a  x 2  x   c a  x 2  x  2 a  a 4a  .  b2 c   4a. 2. b  b 2  4ac b 2  4ac  = a  x+   ³ 4a 4a  2a  2 4ac  b b MinP  4a khi x= 2a VËy :. b) Điều kiện: x ³0 đặt t= x ³0 2. 1  1 1 B t 2  t  t    ³ 4  2 4 1 1 1 MinB  khi x  4 2 hay x= 4 VËy : 2 2 c) §Æt t |3x-1| Þ 9 x  6 x t  1 tacã :. C t 2  4t  5 (t  2) 2  1 ³1 1 VËy : Min C =1 khi t = |3x-1| = 2 Û x 1 hoÆc x= 3 . Bµi 2: a) Cho a < b,t×m gi¸ trÞ nhá nhÊt cña A = |x-a| + |x-b|. b) Cho a < b < c, t×m gi¸ trÞ nhá nhÊt cña B= |x-a| + |x-b| + |x-c| Gi¶i.

<span class='text_page_counter'>(103)</span> a ) áp dụng bất đẳng thức : |x+y| Ê| x |  | y | dấu bằng xảy ra khi và chỉ khi xy ³0 ta cã : A = |x-a| + |x-b| = |x-a| + |b-x| ³| x  a  b  x |b  a x  a b  x ³o Û a £x £c.   VËy : MinA = b-a khi  b) Ta cã |x-a| + |x-c| ³c  a ,dÊu b»ng x¶y ra khi vµ chØ khi a £x £c vµ | x-b| ³o dÊu b»ng x¶y ra khi vµ chØ khÜ x=b Do đó :B = |x-a| + |x-b| + |x-c| ³c  a dấu bằng xảy ra khi và chi khi x=b. vËy MinB = c-a khi x = b 4x  3 2 Bµi 3 . T×m gi¸ trÞ nhá nhÊt , lín nhÊt cña y= x  1. Gi¶i. 4 x 1  ax 2  4 x  3  a  a  x 2 1 x 2 1. C¸ch 1. ta cã 2 Ta tìm a để  ax  4 x  3  a là bình phơng của nhị thức ta phải có : D , 4  a (3  a ) 0 Û a = -1 hoÆc a = 4 4 x 1 x2  4 x  4 ( x  2) 2  1   1  ³ 1 x2 1 x2 1 x2 1. Víi a = -1, ta cã :y= Suy ra : Min y = -1 khi x= -2  4x2  4x  1 (2 x  1) 2  4  £4 x2 1 x2 1 Víi a = 4, ta cã y = 4 + 1  Suy ra : Max y = 4 khi x = 2 4 x 1 Û yx 2  4 x  y  3 0 2 C¸ch 2: Ta cã y = x  1 (1) 3   y = o: (1) cã nghiªm x= 4  y ¹0 : (1) cã nghiÖm khi vµ chØ khi D, 4  y ( y  3) ³0 Û ( y  1)( y  4) £0 Û  1 £ y £4. VËy : Min y = -1 , Max y = 4. Bµi 4. a) T×m gi¸ trÞ nhá nhÊt cña y = (x-1)(x+2)(x+3)(x+6) 6 6 2 2 b) T×m gi¸ trÞ nhá nhÊt cña S = x  y biÕt x  y 1 Gi¶i 2 2 2 2 a) Ta cã : y = ( x  5 x  6)( x  5 x  6) ( x  5 x)  36 ³ 36 VËy Min y = -36 khi x = 0 hoÆc x = -5.

<span class='text_page_counter'>(104)</span> b) Ta cã : S  x 6  y 6  x 2  y 2 = x2  y2. .  . 2. x. 4.  y 4  x2 y 2. .  3x 2 y 2 1  3 x 2 y 2 £1. VËy : MaxS = 1 khi x = 0 , y ±1 hoÆc x ±1 , y = 0 .. Bµi 5 ( x  a )( x  b) x T×m gi¸ trÞ nhá nhÊt cña A= víi x > 0. Gi¶i. 2. x  (a  b) x  ab ab x   a  b x x Ta cã : A= ab ³2 ab áp dụng bất đẳng thức cô si, ta có : x + x do đó: 2 A ³a  b  2 ab ( a  b ). Bµi 6. 2 VËy minA=( a  b ) khix  ab. 2. 2. 2. 2. Cho biÓu thøc M = x  y  2 z  t víi x,y,z,t Î N . H·y t×m gi¸ trÞ nhá nhÊt cña Mvµ c¸c gi¸ trÞ x,y,z,t biÕt r»ng :  x 2  y 2  t 2 =21 (1)  2 2 2  x  3 y  4 z 101 (2). Gi¶i Lấy (1) cộng (2) ta đợc : 2(x2 + y2 + 2z2 + t2 ) - t2 = 122. Do đó : t2 ³61 M = 61 + 2 Þ MinM = 61 khi t = 0 .. Víi t = 0 , tõ (1) suy ra : x2 - y2 = 21 Û (x - y)(x + y) = 21..  x  y 1  x 11 Û  i)  x  y 21  y 10 (lo¹i v× kh«ng tho¶ m·n (2)) .  x  y 3  x 5 Û .  x  y  7 y  2   ii) Thay vµo (2) ta cã z = 4 .. Bµi7. VËy : MinM = 61 khi x = 5 , y = 2 , z = 4 , t = 0 .. a) T×m gi¸ trÞ nhá nhÊt cña A= x  1  y  2 biÕt x+y=4 y 2 x 1  y b) T×m gi¸ trÞ lín nhÊt cña B= x c) T×m gi¸ trÞ lín nhÊt cña C=x+ 2  x. Gi¶i. a) §iÒu kiÖn : x ³1, y ³2, ta cã : A2  x  y  3  2 ( x  1)( y  2). , ¸p dông bÊt d¼ng thóc c« si tacã :.

<span class='text_page_counter'>(105)</span> 2 ( x  1)( y  2) £x  1  y  2. Do đó :. A2 £2 Þ A £ 2 3  x  x-1 = y-2   2 Û   x+y = 4  y 5  2 2 khi. VËy : MaxA = b) áp dụng bất đẳng thức cô si ta có:. ( x  1)  1 x ( y  2)  2 y  ( y  2)2 £  2 2 , 2 2 1 1 1 1 £  Þ max y   khi x 2, y 4 2 2 2 2 2 2 Suy ra : B 2 c Đặt y  2  x ³0( x Ê2), ta có x 2  y . khi đó : 9 1 9 9 1 7 y 2   ( y  )2 £ Þ  khi y  Û x  4 2 4 2 4 C=y+2MaxC 4 ( x  1)1 £. bµi 8 :. Tìm số có hai chữ số sao cho tỉ số giữa số đó và tổng các chữ số của nã cã gi¸ trÞ a) nhá nhÊt : b) lín nhÊt: Gi¶i : Gi¶ sö cã hai ch÷ sè lµ ab(1 £a £9, o £b £9, a, b Î N ), ta cã : ab 10a  b a  b  9a 9   1  b a b a b a b 1 a ab b a ) tØ sè a  b nhá nhÊt khi a lín nhÊt , tøc lµ khi b = 9 vµ a = 1. vËy sè cÇn t×m lµ 19. ab b b) tØ sè a  b lín nhÊt khi a nhá nhÊt , tøc lµ b = 0 c¸c sè cÇn t×m lµ. Bµi 9. 10,20 , ......, 90. a) chøng minh r»ng : i)nếu hai số có tổng không đổi thì tích của chúng lớn nhất khivà chỉ khi hai số đó bằng nhau ii) nếu hai số dơng có tích không đổi thì tổng của chúng nho nhất khi và chỉ khi hai số đó bằng nhau: ¸p dông: 3 3 3 i) t×m gi¸ trÞ lín nhÊt cña A= x (16  x ) víi 0< x  16. ( x  a)2 ii) t×m gi¸ trÞ nhá nhÊt cña B= x víi a,x >0. gi¶i.

<span class='text_page_counter'>(106)</span> a). i) giả sử x,y >0 ta có x+y=k (không đổi). áp dụng bất đẳng thức c«si cho hai sè x,y ta cã: x+y. ³2 xy Þ xy £. k2 4 ,DÊu "="x¶y ra khi. k vµ chØ khi x=y= 2. k2 k khix  y  2 Do đó max(xy)= 4. ii) Giả sử x , y > 0và k = xy (không đổi) , ta có : 2 xy £x  y Þ x  y ³2 k Þ Min( x  y ) 2 k khi x = y b, ¸p dông : i). 3 3 3 3 ta có x  (16  x ) 16 (không đổi) nên tích x (16  x ) lớn 3 3 nhÊt khi vµ chØ khi x 16  x Þ x 2 VËy maxA=16 khi x=2. x 2  2 ax  a 2 a2 a2  x  2a  x x .x vµ x lµ hai sè d¬ng cã tÝch ii) ta cã : 2 a a2 x a 2 x không đổi nên tổng x+ x đạt giá tri nhỏ nhất khi x= B. a2 Û x a x. Bµi 10. VËy min B = 4a khi x = a. 1  x 1 a) T×m gi¸ trÞ lín nhÊt cña y (1  x)(2 x  1) víi 2 1 y x  x  1 víi x>1 b) T×m gi¸ trÞ nhá nhÊt cña c) T×m gi¸ trÞ lín nhÊt cña A=(3-x)(4-y)(2x+3y) víi 0 £x £3, 0 £ y £4. Gi¶i. 1 1 (2  2 x )(2 x  1)  x 1 a) ta cã : y = 2 víi ®iÒu kiÖn 2 th× hai sè 2 - 2x , 2x - 1 lµ. hai số dơng có tổng : (2 -2x)+(2x- 1) =1 (không đổi) nên tích (2-2x)(2x3 1) lín nhÊt khi 2 - 2x = 2x - 1 hay x = 4 3  3 3  1  1     1  khix  4 VËy : Maxy=  4   2  8.

<span class='text_page_counter'>(107)</span> 1 1 1 x 1 b) ta cã víi x>1 th× hai sè d¬ng x-1 vµ x  1 cã tÝch b»ng 1 1 1 2 x  1 x  1 1  x  1 x  1 nªn tæng nhá nhÊt khi x-1 = hay hay x = 2 (v× y x  1 . >1) VËy : Min y=3 khi x=2. c). 1 (6  2 x )(12  3 y )(2 x  3 y ). Ta cã : A = 6 víi 0 £ x £3, 0 £ y £4 th× ba sè. không âm 6 -2x , 12 - 3y và 2x + 3y có tổng bằng 18 (không đổi) nên tÝch cña chóng lín nhÊt khi 6 - 2x =12 - 3y = 2x + 3y = 6  2 x  12  3 y  2 x  3 y 3 (tØ lÖ thøc ) hay. 6 - 2x = 12 - 3y = 2x + 3y = 6 tøc lµ x = 0 , y = 2 . VËy MaxA = 6 khi x = 0 , y = 2 Bµi11 a) t×m gi¸ trÞ lín nhÊt cña S = xyz ( x  y )( y  z )( x  z ) víi x,y,z > 0 vµ x+y+z = 1 4 4 4 b) cho xy + yz + zx =1 T×m gi¸ trÞ nhá nhÊt cña x  y  z .. Gi¶i a) áp dụng bất đẳng thức cô si ta có 1 = x+y+z. ³3 3 xyz Û xyz £. 1 27 (1). 1 dÊu "=" x¶y khi vµ chØ khi x = y = z = 3 ³3 3 ( x  y )( y  z )( x  z ). 2 = (x + y) + (y + z) + (x + z) Þ ( x  y )( y  z )( z  x ) £ S£. 8 27 (2). 8 8 1 khix  y  z  729 Do đó max S = 729 3. Tõ (1) vµ (2) suy ra b) áp dụng bất đẳng thức bunhiacôpski, ta có :. 2 2 2 2 2 2 2 2 2 2 2 1 = (xy + yz + xz) £( x  y  z )( x  y  z ) ( x  y  z ) (1) lại áp dụng bất đẳng thức bunhiacôpski, ta có :. ( x 2  y 2  z 2 ) 2 £(12  12  12 )( x 4  y 4  z 4 ) 3( x 4  y 4  z 4 ) (2) 1 x4  y 4  z 4 ³ 3 Tõ (1) vµ (2) suy ra : 1 ± 3 x 4  y 4  z 4 )  khix  y  z  3 3 VËy Min(. Bµi 12 T×m gi¸ trÞ lín nhÊt vµ nhá nhÊt cña x tho¶ m¶n hÖ sau.

<span class='text_page_counter'>(108)</span>  x  a  b  c 7  2 2 2 2  x  a  b  c 13. Víi a,b,c lµ tham sè. Gi¶i áp dụng bất đẳng thức bunhiacôpski , ta có :.  a  b  c. 2. £ 12  12  12 a 2  b 2  c 2 Û (a  b  c)2 £3(a 2  b 2  c 2 ). . . . (có thể chứng minh công thức này bằng phép biến đổi tơng đơng ). 2 2 2 2 Từ giả thiết ,ta có :a + b + c = 7 - x và a  b  c 13  x do đó.  7  x. 2. £3(13  x 2 ) Þ 4 x 2  14 x  19 £0 Þ 1 £ x £. 5 2. Vậy : x đạt giá trị nhỏ nhất là1 khi a = b = c = 2 5 3 x đạt giá trị lớn nhất là 2 khi a = b = c = 2. Ba× 13 2 2 a) T×m gi¸ trÞ lín nhÊt cña P  x  y  xy  2 x  2 y b) T×m gi¸ trÞ nhá nhÊt cña biÓu thøc : 2 2 M = xy(x - 2)(y + 6) + 12 x  24 x  3 y  18 y  36 Gi¶i a) Ta cã 2 P  2 x 2  2 y 2  2 xy  4 y  4 y 2 2 2 2 = ( x  2 xy  y )  ( x  4 x  4)  ( y  4 y  4)  8 2. =. 8   x  y   ( x  2) 2  ( y  2) 2 £8. P £4. Suy ra : dÊu "=" x¶y ra khi vµ chØ khi x = y = 2 VËy : MaxP = 4 khi vµ chØ khi x = y = 2 2 2 b) Ta cã : M = x( x  2) y( y  6)  12 ( x  2 x)  3( y  6 y )  36. =. x ( x  2) [ y ( y  6)  12]  3( y 2  6 y  12). 2 2 = ( x  2 x  3)( y  6 y  12) 2. 2 Mµ : x  2 x  3  x  1  2 ³2. y 2  6 y  12 ( y  3) 2  3 ³3 Þ M ³6 .DÊu "=" x¶y ra khi vµ chØ khi x = 1, y = -3. VËy : min M = 6 khi vµ chØ khi x = 1, y = -3 Bµi 14 H·y ph©n chia 8 sè : 2 ,3, 4, 5, 6, 8, 9,10 thµnh hai nhãm tuú ý råi lÊy tÝch cuả các số trong mỗi nhóm và gọi A là tổng của hai tích đó .Tìm giá trị nhỏ nhất của Avà chỉ ra các cách chia để số A nhỏ nhất Gi¶i Gọi tích của các số trong mổi nhóm của một cách phân chia nào đó là x , y th× A=x+y víi c¸ch ph©n chia bÊt k× ta cã :.

<span class='text_page_counter'>(109)</span> xy 2.3.4.5.6.8.9.10 28.34.52 ( không đổi) 8. 4. 2. 4. 2. Theo bất đẳng thức cô si : A = x + y ³2 xy 2 2 .3 .5 2 .3 .5 thì giá trị nhỏ 4 2 nhất của Alà 2 .3 .5 1440 khi x=y . để có phân chia sao cho A nhỏ nhất , ta 4 2 thÊy mçi nhãm ph¶i chøa thõa sè 2 ,3 ,5. ta cã c¸ch ph©n chia nh sau. C¸ch 1 2 3. Nhãm 1 2 , 3, 4 ,6 , 5 2 , 4 , 9 , 10 2,5,9,8. Nhãm 2 8 , 9 , 10 8 , 3 , 6 ,5 3 , 4 , 6 , 10. Bµi 15 x( x . 1 1 1 1 1 )  y ( y  )  z ( z  )  t (t  ) £ 4 4 4 4 2 T×m gi¸ trÞ. cho x,y,z,t ³0 tho¶ m¶n : lín nhÊt cña tæng S = x + y + z + t Gi¶i. x2  y 2  z 2  t 2 . Tõ gi¶ thiÕt suy ra : đẳng thức bunhiacôpski , ta có :.  x  y  z t. 2. . £ 12  12  12  12. x. 2. 1 1 ( x  y  z  t ) £ (1) 4 2 ¸p dông bÊt.  y2  z 2  t 2.  Hay :. 1 2  x  y  z  t  £x 2  y 2  z 2  t 2 (2) 4 1 1 1 2  x  y  z t   x  y  z t £ 4 2 Hay : Tõ (1) vµ (2) suy ra : 4 S 2  S  2 £0 Þ  1 £S £2 1 VËy : MaxS=2 khi x = y = z = 2. Bµi 16 cho tø gi¸c låi ABCD cã diÖn tÝch S vµ O lµ ®iÓm n»m trong tø gi¸c 2 2 2 2 sao cho OA  OB  OC  OD 2 S . chøng minh r»ng ABCD lµ h×nh vu«ng cã t©m lµ O Gi¶i Gọi BH là đờng cao của tam giác ABC , ta có : 2 S ABC =OA.BH. nhng OA2  OB 2 2 S AOB £OA.OB £ 2. BH £BO nªn : (BĐT cô si) .Do đó 1 2 2 S AOB £ 4 (OA  OB ), dÊu b»ng x¶y khi vµ chØ khi OA  OB vµ OA=OB. 1 £ (OB 2  OC 2 ); S BOC 4 T¬ng tù : 1 2 2 S COD £ 4 (OC  OD ) ; C. B. O. H. A. D.

<span class='text_page_counter'>(110)</span> S. AOD. 1 £ (OA2  OD 2 ) 4 S ABCD S AOB  S BOC  S COD  S DOA. OA2  OB 2  OC 2  OD 2 £ 2. VËy. 2. 2. 2. 2. Hay : 2S £OA  OB  OC  OD , dÊu b»ng x¶y ra khi vµ khi vµ chØ khi . . . . 0. OA=OB=OC=OD vµ AOB BOC COD DOA 9O Tøc lµ ABCD lµ hình vuông và O là tâm của hình vuông đó Bµi 17 a b  1, Cho trớc a , b > 0 và x , y> 0 thay đổi sao cho x y tìm x , y để x. + y ®at gi¸ trÞ nhá nhÊt . Gi¶i. ¸p dông bÊt ®¨ng thøc bunhiac«pski ta cã :. . 2. x  y. x+y=. 2. . 2 2 2      a b a b          ³ x  y     x x y   y    . 2 Hay : x  y ³( a  b ) , dÊu "=" x¶y ra khi. y x x y x y  Û    a  b, a b a b a b x y.  min( x  y )  VËy x a. tøc lµ khi.  b. a  b , y  b ( a  b ). a. 2. khi. x a. . . a  b , y  b( b  a). Bµi tËp t¬ng tù 1. T×m gi¸ trÞ lín nhÊt cña : y ax 2  bx  c ( a < 0 ) 2. T×m gi¸ tri nhá nhÊt cña: y | x  a |  | x  b |  | x  c |  | x  d | víi a < b < c < d. 3 .T×m gi¸ trÞ nhá nhÊt cña :. y  x100  10 x10  10.. 4.T×m gi¸ trÞ nhá nhÊt , lín nhÊt cña : y. x2  x 1 x2  x 1. Hớng dẩn giải và đáp số 4ac  b 2 . 4a 1: 2: min y c  d  a  b max y .

<span class='text_page_counter'>(111)</span> 3: min y 1 1 min y  , max y 3 3 4:. bÊt ph¬ng tr×nh A KiÕn thøc c¬ b¶n .. A B Û  B AB.  B  0   A ³0 A  B Û   B ³0    A  B 2 . 2 . A  B Û 0 £A  B. AB A BÛ  A B .. Bµi 1 Gi¶i bÊt ph¬ng tr×nh 2 a ) x  2x  3  0. B . C¸c bµi to¸n b). 2 x  3 £5. 1 1  d ) x 1 x. c ) 2x  1  x. Gi¶i. 2. a ) x  2x  3  0 Û  1  x  3 2 x  3 £5 Û  5 £2 x  3 £5 Û  1 £ x £4 1 x³ : 2 c ) ®iÒu kiÖn :. b). 2. 2 x  1  x Û 2 x  1  x 2 Û  x  1  0 Û x ¹1. 1 x ³ , x ¹1 2 VËy 1 1 1  Û  0 Û x( x  1)  0 Û  1  x  0 x  1 x x ( x  1) d). Bµi 2 Gi¶i bÊt ph¬ng tr×nh x a) . b). 2. 2.  2 x  7  7 x2  2 x  8  7  0. x  x  2. . 2. . .  x  4  £5. x 1 x  £2 c ) x x 1 2. a ) §Æt t  x  2 x  7 ta cã :. Gi¶i.

<span class='text_page_counter'>(112)</span>  x2  2 x  7  0  t 2  7(t 1)  1  0 Û t (t  7)  0 Û 0  t  7 Û  2  x  2x  7  7  x  1 2  6  0 Û Û 2 x0  x  2  .x  0 2. b) Ta cã :. x( x  2) 2 ( x  4) £5 Û  x 2  4 x   x  2  £5. đặt. t  x  2 . 2.  t  4  t £5 Û t. ( t ³0 ), ta cã : 2.  4t  5 £0 Û  t  1  t  5 £0 Û t £5. (v× t +1> 0 ). 2. Û  x  2  £5 Û x  2 £ 5 Û  5  2 £ x £ 5  2 x , x  1 ta cã : c ) §Æt  t 1 1 t 2  2t  1 t  £2 Û £0 Û  t t t  0 x Û 1 Û 1 0 x 1  t =1 , v« lÝ x Û  0 Û  1 x  0 x 1  t<0 t. Bài 3 xác định m để bất phơng trình x  m Ê2 x  2 có nghiêm. Gi¶i. 2. đặt t  x  2 ³0 ị x t  2 . ta có 2. t 2  2  m £2t Û  t  1 £m  3. bÊt ph¬ng tr×nh cã nghiÖm th× m  3 ³0 Û m ³ 3 víi m ³ 3 : (1) Û t  1 £ m  3 Û 1  m  3 £t £1  m  3 cã nghiÖm t ³0. VËy m ³ 3 Bài 4 xác định m lớn nhất để x(x+1)(x+2)(x+3) ³m thoả với mọi x. Gi¶i. Ta cã x(x+1)(x+2)(x+3) ³m. x. 2. 2. 2.  3x   2  x 2  3x  ³m Û  x 2  3 x  1 ³m  1. NÕu (1) tho¶ m¶n víi moi khi m = -1 x Ngîc l¹i , víi m=-1 th× . 2. x. . 2.  3 x  1 ³0 4. Bµi 5 Gi¶i hÖ bÊt ph¬ng tr×nh.  3± 5 2 ta cã m £ 1 suy ra m lín nhÊt. 2.  x  y £1 (1)   5 3   x  y ³1 (2). víi mäi x.

<span class='text_page_counter'>(113)</span> Gi¶i 4.   x £1   1 £ x £1 Þ   2 y £1  1 £ y £1   Tõ (1) suy ra nh©n (2) víi -1 råi céng víi (1) ta ®4 5 2 3 4  x  x    y  y  £0 Û x  1  x   y 2  1  y  £0. îc :.   x 1  4   x (1  x ) 0  y 0 Û 2 Û   x 0   y (1  y ) 0    y 1. VËy bÊt ph¬ng tr×nh cã nghiÖm ( 1 , 0 ) vµ ( 0 , 1) C Bµi tËp ph¬ng tr×nh 1. Gi¶i c¸c bÊt ph¬ng tr×nh sau : a). 4 x 2  2  2 x 1  3 £0 3. b). 2 x  x 1 5 2x  5  7  4x. 2. c ) x  2x  x  2  0 d) 2. Gi¶i vµ biÖn luËn ph¬ng tr×nh : mx - 1 > m - x 4. 2. 3. Gi¶i bÊt ph¬ng tr×nh : x  8 x  16 ³2  x 4. Xác định m để hệ phơng trình sau có nghiệm (x ; y) với x + y lớn nhất :.  x  m2 y £m   2  m x  y £m. ..  x  m £m  x  1  m £2m 5. Định m để hệ sau có nghiệm duy nhất : . D Hớng dẫn giải và đáp số. 1 2 1. a) b) XÐt c¸c trêng hîp : x  0;0 £ x £1; x  1. x3  2 x 2  x  2  x 2  1  x  2  . 2. 4 x 2  4 x  1 £0 Û  2 x  1 £0 Û x . c). . . A  B Û A2  B 2  0.. d) 2. (m + 1)(x + 1) > 0 .  m > - 1; x > -1 .  m<-1;x<-1.  m = - 1 ; V« nghiÖm .   x  2   x  3 ³0  x 2  4 ³2  x x  4 ³2  x Û  2 Û Û x  2   x  1 £0   x  4 £ x  2   3. 2.  x £ 3  x ³ 1  .. 2m x0  y0 £ £1. 1  m2 4. Giả sử hệ có nghiệm (x0 ; y0) . Cộng lại ta đợc :. Suy ra x0 + y0 lín nhÊt b»ng 1 khi m = 1 ..

<span class='text_page_counter'>(114)</span> 5. Điều kiện m ³ 0 . Hệ đợc viết lại :   m £ x  m £m Û    2m £ x  1  m £2m. 0 £x £2m  1  3m £x £m  1.  2m 1  3m 1  0 m  1 Û m  5 . HÖ cã nghiÖm duy nhÊt khi vµ chØ khi : .

<span class='text_page_counter'>(115)</span>

<span class='text_page_counter'>(116)</span>

<span class='text_page_counter'>(117)</span>

<span class='text_page_counter'>(118)</span>

<span class='text_page_counter'>(119)</span>

<span class='text_page_counter'>(120)</span>

<span class='text_page_counter'>(121)</span>

<span class='text_page_counter'>(122)</span>

<span class='text_page_counter'>(123)</span>

<span class='text_page_counter'>(124)</span>

<span class='text_page_counter'>(125)</span>

<span class='text_page_counter'>(126)</span>

<span class='text_page_counter'>(127)</span>

<span class='text_page_counter'>(128)</span>

<span class='text_page_counter'>(129)</span>

<span class='text_page_counter'>(130)</span>

<span class='text_page_counter'>(131)</span>

<span class='text_page_counter'>(132)</span>

<span class='text_page_counter'>(133)</span>

<span class='text_page_counter'>(134)</span>

<span class='text_page_counter'>(135)</span>

<span class='text_page_counter'>(136)</span>

<span class='text_page_counter'>(137)</span>

<span class='text_page_counter'>(138)</span>

<span class='text_page_counter'>(139)</span>

<span class='text_page_counter'>(140)</span>

<span class='text_page_counter'>(141)</span>

<span class='text_page_counter'>(142)</span>

<span class='text_page_counter'>(143)</span>

<span class='text_page_counter'>(144)</span>

<span class='text_page_counter'>(145)</span>

<span class='text_page_counter'>(146)</span>

<span class='text_page_counter'>(147)</span>

<span class='text_page_counter'>(148)</span>

×